You are on page 1of 82

NP1 (81-100)

SITUATION: Twenty years after Mrs. Salazar was first diagnosed with rheumatoid arthritis, she is admitted for a right total hip
replacement. She has experienced severe right hip pain that has not responded to treatment for several years, and has had
increasing difficulty moving about because of damage to the right hip joint.
81. Preoperative teaching for Mrs. Salazar should include
a. Isometric exercises of the quadriceps and gluteal muscles
b. Instructions on the necessity for keeping the right leg perfectly straight after surgery
c. The need to flex the involved hip postoperatively to maintain mobility
d. The avoidance of aspirin for 4 days prior to surgery
82. Which of the following should the nurse consider to be most significant if noted when checking Mrs. Salazar three days
postoperatively?
a. Pain in the operative site c. Swelling of the operative sites
b. Pain and tenderness in the calf d. Orthostatic hypotension
83. The physical therapist orders exercises of Mrs. Ss right hip, knee, and foot to gradually increase range of motion to the right
hip. The nurse can best assist Mrs. S by
a. Administering an analgesic before the exercises
b. Stopping the exercises if Mrs. Salazar experiences pain
c. Performing the exercises for Mrs. Salazar
d. Observing Mrs. Salazars ability to perform the exercises
84. In the post-operative phase. Mrs. Salazar should be instructed to avoid
a. Adduction of her right leg c. Abduction of her right leg
b. Bearing any weight on her right leg d. The prone position in bed
85. The nurse and Mrs. Salazar plan for her rehabilitation. Mrs. Salazar asks the nurse, What do I have to do in therapy?
Which reply by the nurse most accurately describes the task of the patient in rehabilitation?
a. Follow the instructions of the rehabilitation team
b. Regain some function that was lost
c. Prevent further loss of your ability to function
d. Learn to deal realistically with your disability

SITUATION: Roy, is a 66 year old retired engineer is admitted at the emergency department of Isulan Provincial Hospital
because of chest pain and shortness of breath. History revealed previous history of chest pain and had been diabetic for almost
ten years.
86. The doctor orders Nitroglycerin tablet. Which should the nurse instruct the patient to do?
a. Chew the tablet until it is dissolved c. Place the tablet between his cheek and gums
b. Swallow the tablet with one glass of water d. Put the tablet under the tongue until it is absorbed
87. Which of the following controllable risk factors for coronary artery disease appears most closely linked to the development of
the disease?
a. Age c. High cholesterol level
b. Medication usage d. Gender
88. Roys condition did not improved and the doctor suspects of myocardial infarction. The nurse started on IV morphine with
the intended effect of which of the following?
1. Reduce myocardial oxygen consumption
2. Prevents ventricular remodeling
3. Promotes reduction in respiratory rate
4. Reduces blood pressure and heart rate
5. Reduces anxiety and fear=
a. 1, 2 and 5 b. 1, 4 and 5 c. 1, 2, 3 and 4 d. 1, 2, 4
and 5
89. Roy receives reperfusion therapy in the coronary care unit. He is now receiving an IV infusion of Heparin at 2000 units per
hour. The dilution is 25,000 units in 500 ml of D5Water. How many ml per hour will Roy receive?
a. 24 ml /hour c. 40 ml/hour
b. 32 ml/hour d. 80 ml/hour
90. The nurse is performing the 12-lead ECG on Roy with chest pain. Because the positioning of the electrodes is crucial, how
does the nurse place the ECG components?
a. Four leads are placed on the limbs and six are place on the chest.
b. The negative electrode is place on the left arm and the positive electrod is placed on the right leg.
c. Four leads are placed on the limbs and four are placed on the chest.
d. The negative electrode is placed on the right arm and the positive electrode is placed on the left leg.

SITUATION: Mang Remy is a taxi driver who slipped on a wet floor at work and landed on his buttocks. He experienced
immediate severe lower back pain with pain radiating to his right buttock. He rested at home for two weeks with slight relief
hence the consult at the OPD.
91. Mang Remy was treated with rest and drug therapy. An important nursing intervention during this time is to
a. Monitor neurologic status c. Encourage foot and leg exercise every 4 hours
b. Assess the use of body mechanics d. Supine position with knees flexed and head elevated
92. The nurse teaches Mang Remy recovering from an episode of acute low back pain to
a. Perform daily exercise as a lifelong routine
b. Sit in a chair with the hips higher than the knees
c. Avoid occupations in which the use of the body is required
d. Sleep on the abdomen or on the back with legs extended
93. To help prevent Mang Remy with low back pain from assuming a sick role behavior, the nurse
a. Teaches him alternative pain control c. Discusses the need for lifestyle adjustment
b. Encourage activity within limitation d. Moniotr for decreased muscle strength
94. A laminectomy with spinal fusion was done for Mang Remy after months of conservative treatment. Post operatively which
of the following findings is of most concern to the nurse?
a. Paralytic ileus c. Greater pain at the incision site
b. Urinary incontinence d. Leg and arm sensation unchanged from preop status
95. Before repositioning Mang Remy to the side after lumbar laminectomy, the nurse
a. Raises the head of the bed 30 degrees c. Have him grasp the side rails on opposite side
b. Have him flex the knee and hips d. Places pillow between his leg

SITUATION: Janice is a 37 year old mother of three children. During her last self breast examination, which she only does
sporadically, she noticed a lump in her upper outer quadrant of her left breast. Mammography and biopsy revealed medullary
carcinoma of the breast.
96. When teaching Janice about breast self-examination (BSE), the nurse will instruct the patient that
a. BSE will reduce the risk of dying from breast cancer.
b. Performing BSE right after the menstrual period will improve comfort
c. BSE should be done daily while taking a bath or shower
d. Annual mammograms should be scheduled in addition to BSE
97. Which of the following characteristic of breast mass may indicate malignancy?
a. Smooth consistency c. Irregualr shape
b. Tissue mobility d. Tenderness
98. Janice is with positive biopsy for breast cancer is considering whether to have a modified radical mastectomy or breast
conservation surgery (lumpectomy) with radiation therapy. Which information should the nurse provide?
a. The postoperative survival rate for each is about the same, but there is a decreased rate of cancer recurrence after
mastectomy
b. The hair loss associated with post-lumpectomy chemotherapy is not acceptable to some patients.
c. The treatment period for the mastectomy is shorter, and breast reconstruction can provide a normal-appearing breast.
d. The lumpectomy and radiation will preserve the breast, but this method can cause changes in breast
sensitivity.
99. Janice decided to have a modified radical mastectomy which include axillary node removal and immediate reconstruction.
The nurse knows that the axillary node will be removed in order to

a. Prevent metastasis c. Facilitate breast reconstruction


b. Facilitate postoperative recovery d. Provide prognostic information
100. While on Tamoxifen therapy, Janice should be counselled to have an annual Papanicolau smear and pelvic examination
because
a. Will have vaginal dryness c. High risk for endometrial cancer
b. Planning to resume sexual activity d. High risk for cervical cancer

NP3 (1-55)
SITUATION:Winnie is a patient with peripheral vascular disease in the lower extremities. She works as a teacher in a big
university in Kabacan and been complaining of discoloration on her lower extremities with mild pain on prolonged standing. She
had been hypertensive for two years.
1. The nurse is assessing the loweer extremity of Winnie with peripheral vascular disease. During the assessment, the nurse
should expect to find which of the following clinical manifestation of PVD?
a. Hairy legs c. Pink, cool skin
b. Mottled skin d. Warm, moist skin
2. Which of the following statement about the use of garlic as a therapy for Winnies condition?
a. It promotes blood pressure reduction through vasodilation of vessels
b. Lowers cholesterol by interfering with absorption of substance in the GI tract
c. Increases platelet aggregation and promotes clotting
d. Inhibits synthesis of prostaglandin and promotes an anti-inflammatory effect
3. Which intervention would be appropriate for the nurse to include when discussing foot care to Winnie with peripheral
vascular disease?
a. Use heating pads to improve circulation on the leg
b. Cut off all corns and calluses
c. Wash the feet with warm water and mild soap
d. Sit with legs crossed to reduce swelling
4. Which among the following is the best measurement and indicator of tissue perfusion?
a. Systolic blood pressure c. Diastolic blood pressure
b. Mean arterial blood pressure d. Pulse pressure
5. Reduction of sodium intake is one type of nonpharmacologic treatment for hypertension. What is the expected effect of this
treatment?
a. Promote weight loss by dieresis c. Stimulate parasympathetic nervous system
b. Increases serum osmolality d. Decreases vascular fluid volume
SITUATION: The maintenance of a good nursing care is rooted on a sound knowledge on medical surgical care nursing. Nurses
in various setting play an integral part in the maintenance of homeostasis that any disturbances brought about by body system
disruption will require outmost attention to ensure continuity of life.
6. Lester is having difficulty breathing is admitted to the hospital. The best approach for the nurse to use to obtain a complete
health history is to
a. Use the doctors history to gather subjective data
b. Obtain subjective data about the patient from the family members
c. Delay data collection and focus only on the physical examination
d. Schedule several short sessions with the patient to gather data
7. Lester is an emphysematous patient becomes dyspneic as he walks down the hall. What should be the nurse first
response?
a. Instruct him to lean against the wall and bend shoulder slightly forward
b. Help him walk back to the room and have him lie down to rest
c. Stay with the patient and call the aide to bring portable oxygen tank
d. Instruct him to sit down and lean forward with elbows on his knees
8. Lester has fever and chills and difficulty on breathing on exertion. Based on patients history and present status, the priority
nursing action is
a. Checking for capillary refill c. Suctioning secretions from airway
b. Encouraged increased fluid intake d. Administering high concentration of oxygen
9. Lester developed spontaneous pneumothorax from a ruptured subpleural bleb of his emphysematous lungs. What
physiologic effect of spontaneous pneumothorax should the nurse include in the teaching plan for the patient?
a. Heart and great vessel shift to the affected side
b. Greater negative pressure within the chest cavity
c. Inspired air will move from the lungs into the pleural space
d. Other lung will collapse if not treated immediately
10. What clinical indicator should the nurse expect to identify when assessing an individual with spontaneous pneumothorax?
a. Hematemesis c. Increased thoracic motion
b. Mediastinal shift toward involved side d. Unilateral chest pain

SITUATION: Mrs. Bustamante, 72 years old, was admitted to the medical ward with the following complaints:
enlarged abdomen, loss of appetite, dry skin, and pallor. She appears to be confused and agitated. She has an
intravenous drip of D5LR at 800 mL level running at 20 drops per minute. Vital signs are within normal range but the
client appears anxious and disoriented and constantly asks where her family members are.
11. Given the above clients symptoms the nurses immediate concern for the client is to:
A.Monitor vital signs every 4 hours
B.Promote clients safety and reduce anxiety by frequent orientation to her location
C.Hydrate client adequately by encouraging and measuring intake and output
D.Ensure the presence of a family member as caregiver

Rationale:
Option A is incorrect.
This is not an immediate concern because it has a time frame and monitoring the vital signs is done routinely.
Option B is correct.
For questions involving trauma scene, think safety first!

Source: NCLEX-RN Questions and Answers Made Incredibly Easy!


-Always think safety first.

Source: p.816 Davis's NCLEX-RN Success by Sally L Lagerquist

Management of patients with delirium continues to focus on ensuring safety from behavioral disturbances while
simultaneously assessing for a probable etiology and definitive treatment. Safety can best be addressed by
combining environmental, behavioral and pharmacologic means.

Source: p.62 American Psychiatric Association Practice Guidelines by American Psychiatric Association

Option C and D are both incorrect.


These options do not provide proper and relevant way of addressing the anxiety as well as the disorientation of the
client.

12. The clients age and condition warrants that the health care team protects Mrs. Bustamante from possible injury
to self. The nurse ensures clients safety through:
A.Use of wrist restraints to prevent the client from getting up without assistance
B.Providing assistance to the client when getting in and out of bed or chair
C.Keeping side rails up to discourage client from getting out of bed
D.Maintaining client on bedrest to reduce ambulation

Rationale:
Option A is correct.
-Restraining measures are used to prevent:
The patient from falling out of bed, a chair or wheelchair because of disorientation
The interruption of therapy, such as traction, intravenous infusions, nasogastric tube feeds or urinary
catheters.
The removal by disoriented patients of life support measures such as endotracheal tubes or the
prevention of self-injury such as scratching wounds.
Disoriented or aggressive patients from injuring or harming other patients

Source: Practical Guide for General Nursing Science By Magda Mulder


Restraint may be imposed to ensure immediate physical safety of the patient, a staff member, or others and must be
discontinued at the earliest possible time.

Restraints are used in the healthcare setting primarily in two general situations:
(1) violent and/or destructive situations when the patient has demonstrated or poses an imminent danger to themself
or another, and (2)disruption of therapy or nonviolent, nondestructive situations. If a patient is harmful to self or
another and cannot be managed using de-escalation techniques, restraints may be appropriate.

Source: Behavioral Emergencies for the Emergency Physician edited by Leslie S. Zun

Option B is incorrect.
-The nurse teaches client to request assistance for activities such as getting out of bed or going to the bathroom. If
the clients cannot request assistance, they require close supervision to prevent them from attempting activities they
cannot perform safely alone.

Source: p.446 Psychiatric-mental Health Nursing by Sheila L. Videbeck

Option C is incorrect.
-Clients with delirium has a disturbance of consciousness raising the side rails will not assure that he will not injure
himself.

. Option D is incorrect.
-Delirium has a disturbance of consciousness and people with delirium are always agitated and cannot
be placed put to bed.

13. The evening of admission Mrs. Bustamante becomes very agitated and restless, pulls out her IV drip, rips her
gown and scratches anyone who approaches her. The doctor was notified and physical restraint is ordered along
with Benadryl 50mg IM stat. When a client is on physical restraints, your priority nursing intervention would be to:
A.Monitor the clients vital signs twice every shift
B.Release the client wrist restrain once every shift for feeding purposes
C.Release the clients restraints every two hours but assess hourly
D.Assess the client every hour but keep restraints tied to 4 extremities

Rationale:
Option A is incorrect.
-Monitoring client's vital signs should be done every 15 minutes.
Option B and D are both incorrect.
-Restraints should be released every 2 hours.

Source: p.341 Fundamentals of Nursing: Content Review Plus Practice Questions by Patricia M Nugent,
Barbara A Vitale

Option C is correct.
-Remove restraint every 2 hours to perform a neurovascular assessment and put the entire extremity
through full range-of-motion(ROM)exercises.
-Remove the restraint every 2 hours and assess skin integrity.

Source: p.341 Fundamentals of Nursing: Content Review Plus Practice Questions by Patricia M Nugent,
Barbara A Vitale

-Assess and assist the restrained patient minimum of every 15 minutes, including injuries caused by the
restraint, nutrition, hydration, circulation, ROM, vital signs, hygiene, elimination, comfort and physical
and psychosocial status. Also assess whether the patient is ready to have restraints discontinued.

Source: p.443 Perfecting Clinical Procedures edited by Lippincott Williams & Wilkins

14. What behavior or action of Mrs. Bustamante will alert you that she may be experiencing delirium?
A.Daytime sleepiness and night time incontinence
B.The client becomes confused within 24 hours from admission
C.Depression alternating with periods of cheerfulness
D.Depression and inability to get out of bed to do activities of daily living

Rationale:
. Option B is correct.
-A delirium is defined as "a disturbance of consciousness and a change in cognition that develop over a
short period of time."(APA,2000,p.135)
-Delirium usually has an acute onset, from hours to days, and fluctuates throughout the day, with
periods of lucidity and awareness alternating episodes of acute confusion, disorientation and perceptual
disturbances.
Source: p.112 Lippincott's Manual of Psychiatric Nursing Care Plans by Judith M. Schultz, Sheila L.
Videbeck

. Option A is incorrect.
Sleep disorders causes nighttime incontinence, nocturnal confusion, hallucinosis and daytime sleepiness.
Source: p.73 Parkinson's Disease and Movement Disorders edited by Joseph Jankovic, Eduardo Tolosa

. Option C is incorrect.
Bipolar disorder is frequently marked by periods of inconsolable sadness and despair(major depression) alternating
or intermixed with periods of extreme cheerfulness.
Source: Mental Disorders, Medications, and Clinical Social Work by Sonia G. Austrian

. Option D is incorrect.
The psychopathology of depression revolves around the impact that the depression has on the patient's activities of
daily living.

Source: Professional Paramedic, Volume II: Medical Emergencies, Maternal Health by Richard Beebe,
Jeffrey Myers

15. When Mrs. Bustamante shows combative behavior against hospital personnel, your most appropriate initial action
would be to:
A.Attempt to talk soothingly, orient her to her surroundings and stay with her
B.Refer to physician to order physical restraints
C.Obtain an order for medications to reduce combative behavior and calm her down
D.Obtain a referral to a mental health psychiatric facility

Rationale:
Option A is correct.
-Speak to client in a calm manner in a clear voice; use simple sentences.
Provide orienting verbal cues when talking with the client.
Source: p.446 Psychiatric-mental Health Nursing by Sheila L. Videbeck

Option B is incorrect.
-The use of restraints as an emergency measure is taken primarily as a last resort to protect a patient from harm.
Source: p.102 CliffsTestPrep: NCLEX-RN by American BookWorks Corporation
-Restrain is a last resort.
Source: p.5 Meeting Special Needs: A practical guide to support children with Attention Disorder by
Selena Ledgerton Cooper

Option C is incorrect.
-If physical restraint won't help, a chemical restraint is done to subdue agitated or confused client.
Chemical restraint is the use of medicationto control patients or manage their behavior.
Source: p.177 Psychiatric Nursing: Contemporary Practice edited by Mary Ann Boyd

Option D is incorrect.
-Delirium is a temporary condition and it is due to medical conditions and not because of psychological conditions.

SITUATION: Clients who have alteration in fluid volume require immediate attention
because of the subsequent health problems these may create. The nurses actions
should address these problems.
16. A nurse assesses the clients during the nursing around. She observes that some
clients are at risk for fluid volume deficit. Among these clients, the nurse should prioritize
giving care to:
A. Mr. De Arroyo, 53, who is experiencing polyuria due to Diabetes Mellitus
B. Mr. De Corona, 69, who has diarrhea and is currently restless
C. Mr. De Enrile, 17, who refuses and spits out food and drinks
D. Mr. De Santiago, 48, who is taking diuretics and has fluid restrictions
Rationale:
Option A is incorrect.
Patient with Diabetes Mellitus has 3 Ps: Polyphagia, Polyuria and Polydipsia.
Polyuria is a characteristic of a patient with DM.
Option B is correct.
If the dehydration is severe, the client may also be restless, confused and complaining
of thirst.
Source: p.541 Lippincott's Review for NCLEX-RN by Diane McGovern Billings
Option C is incorrect. Patient has no presenting symptoms of FVD.
Option D is incorrect. Diuretics can cause water loss but order for this medication can
be cancelled by the physician.

17. Mrs. Conchita Kurapika, 62, has generalized edema and is on diuretics. The easiest
and accurate way to assess the clients response to her medical treatment would be
through:
A. Determining plasma osmolality C. Measurement of pedal
edema
B. Weight measurement D. Examination of serum
sodium level
Rationale:
Option A is incorrect.
Edema occurs when there is too much fluid in the interstitial compartment. It may be
localized or generalized. The causes of edema at the capillary level are increased
capillary hydrostatic pressure, increased interstitial fluid osmotic pressure, blockage of
lymphatic drainage and decreased capillary osmotic pressure.
Source: p.528 Pathophysiology by Lee-Ellen C. Copstead-Kirkhorn, Jacquelyn L.
Banasik

Edema occurs when there is a change in the capillary membrane, increasing the
formation of interstitial fluid or decreasing the removal of interstitial fluid. Sodium
retention is a frequent cause of the increased ECF volume.
Source: p.275 Brunner & Suddarth's Textbook of Medical-surgical Nursing, Volume 1
edited by Suzanne C. O'Connell Smeltzer, Brenda G. Bare, Janice L. Hinkle, Kerry H.
Cheever

Option B is correct.
Daily measurement of body weight must closely reflects total body fluid volume.
Source: p.317 Medical Surgical Nursing by Basavanthappa

Option C is incorrect.
Pedal edema is just part of generalized edema and does not measure the medical
treatment that has been done accurately.

Option D is incorrect.
Hyponatremia in edema forming states can be difficult to treat. As a general rule,
patients with hyponatremia and edema should be fluid restricted and hypotonic fluids
should not be given.
Source: p.746 Pediatric Critical Care Study Guide: Text and Review edited by Steven
E. Lucking, Frank A. Maffei, Robert F. Tamburro, Neal J. Thomas

18. CJ has a renal problem and is on fluid restrictions. He complains of extreme thirst.
To provide some comfort to the client, appropriate nursing interventions include the
following, except:
A. Give ice chips instead of drinking water C. Keep ice chips in the
mouth
B. Offer antiseptic mouthwash D. Allow frequent oral
care
Rationale:
Option A,C,D are all incorrect.
Maintain fluid restrictions according to physician order. Promote oral health and comfort
by completing oral care and providing ice chips.
Source: p.551 Medical Surgical Nursing: An Integrated Approach by Lois White, Gena
Duncan, Wendy Baumle

Option B is correct.
The use of mouthwash can further reduce harmful bacteria in the mouth.
Source:p.329 Lippincott's Textbook for Nursing Assistants: A Humanistic Approach by
Pamela J. Carter

19. Nokare Kagas, 25 years old, has gastroenteritis and frequent diarrhea. He is put on
intravenous fluids. The nurses assessment of Mr. Kagas includes the following, except:
A. Bradycardia C. Decreased urine output
B. Increase in temperature D. Decreased blood pressure

Rationale:
Option A is correct.
Tachycardia, is one of presenting signs of fluid volume deficit.
Fluid volume deficit yields an increased pulse rate with thready quality, decreased BP,
fever and an increased rate and depth of respirations.
Source: p.253 NCLEX-RN For Dummies by Patrick R. Coonan

Option B is incorrect.
Gastroenteritis is an infection and can cause increase in temperature.
Gastroenteritis attacks the intesting and cause the following signs and symptoms:
Watery diarrhea
Abdominal cramps and pain
Nausea,vomiting or both
Occasional muscle aches or headache
Low-grade fever
Source: http://www.mayoclinic.org/diseases-conditions/viral-
gastroenteritis/basics/symptoms/con-20019350

Option C and D are both incorrect.


Fluid volume deficit yields an increased pulse rate with thready quality, decreased BP,
fever and an increased rate and depth of respirations.
Source: p.253 NCLEX-RN For Dummies by Patrick R. Coonan

20. Mr. Sirangkalsada, 48 years old, was taken to the emergency room suffering from
dehydration. He was exercising vigorously, jogging outdoors in the heat of the sun. After
his treatment, the nurse gives health instruction to the client to avoid reoccurrence of
dehydration during exercise. Which of the following statements by the client indicates
that the nurse needs to give more instructions to the client?
A. Wearing thick sweat shirts during summer time to lose weight is not a good
idea.
B. While exercising, I will drink fluid, when I get thirsty.
C. I have to avoid exercising outdoors when it is extremely hot and humid.
D. I will drink cool water before starting to exercise and additional 150ml every 15
minutes thereafter.

Rationale:
Option A is incorrect.
On sunny days, we use umbrella and fan; we wear thin clothing:sleeveless dresses,
sandos, shorts and bathing suits,
Source: p.144 Reading Wonders 6 Teacher's Manual 1st Ed. 2006 by Ulit, Enriqueta Et
Al

Wear proper clothing. Wear porous clothing that allows air to pass through and cool
your body. Also wear light-colored clothing; lighter colors reflect the sun's heat, whereas
darker colors absorbs it.

Source: p.202 Health for Life by McConnell, Karen, Corbin, Charles, Corbin, David,
Farrar, Terri

Option B is correct.
Feeling thirsty is not the best indicator of your body's water needs, because thirst occurs
after your body is already dehydrated. Also, your thirst is usually satisfied even before
your body's water supply is fully replaced. This means that during workouts, you should
drink water even if you do not feel thirsty.
Source: http://www.texasheart.org/HIC/Topics/HSmart/hydrate.cfm

Option C is incorrect.
Exercising vigorously in hot and humid weather can be challenging and even
dangerous. But you can safely exercise in hot weather if you take the proper
precautions. One of the most important things to do is to stay hydrated and decrease
your exercise intensity on very hot days. Keeping your body hydrated during exercise
helps replace the water lost from sweating and prevents fatigue and poor physical
performance.
Source: http://www.texasheart.org/HIC/Topics/HSmart/hydrate.cfm

Option D is incorrect.
Your drink up routine:
Drink at least 150 ml every 15 minutes during exercise.
Source:http://www.deakinburwood.ymca.org.au/Media/docs/Drink-Up-Poster--Smart-
Play-4293e8e7-361f-434f-8b9a-7e778ec2ca6f-0.pdf

SITUATION: Twenty houses were burned in a squatters area in Pasay due to faulty electrical wirings.
Mang Isko, an informal settler in the said locale, obtained severe burns on his body and was rushed to
the ER

21. Mang Isko sustained severe burns of the face, neck, anterior chest and both arms and hands, using
the rule of nines. Which of the following is the best estimate of total body-surface area burned?
A. 18% B. 22% C. 31% D.
40%
22. Based on Mang Iskos total body surface burned and his body weight of 67 kg, which of the following
total amount of fluid is to be given to Mang Isko for fluid replacement in the first 24 hours?
A. 8,300 ml B. 6,400 ml C. 4,150 ml D.
7,700 ml
23. Nursing care planning is based on the knowledge that the first 24-48 hours of post-burn are
characterized by:
A. An increase in the total volume of intracranial plasma
B. Fluid shift from interstitial space
C. Excessive renal perfusion with diuresis
D. Fluid shift from intravascular space to the interstitial space
24. Since Mang Isko has severe burns on the upper torso, which of the following nursing action would be
a primary concern?
A. Debriding and covering the wounds
B. Frequently observing for hoarseness, stridor, and dyspnea
C. Administering antibiotics
D. Establishing a patent IV line
25. Which of the following cluster of symptoms would the nurse notes when a client is suffering from deep
partial-thickness burn?
A. Tingling and hyperesthesia C. Pain free and shock
B. Pain, hyperesthesia, sensitivity to cold air D. Hyperesthesia and pain that is
soothed by cooling

SITUATION: Nurse Kenneth is caring for Jonathan, 40 year old 3 rd day post bowel resection, NPO with
D5LR IV 1000 ml at 125 ml/hr. Laboratory findings show a hemoglobin level of 8 g/dl and hematocrit of 30
%.
26. During the physicians rounds, Dr. Lorenzo made the following orders:
-Gentamycin 80 mg IV piggy back in 50 ml D5 Water over 30 minutes
-Ranitidine 50 mg IV in 50 ml D5 water piggy back in 30 minutes
-Packed red blood cells (RBC) 250 ml to run for 3 hours
How many milliliters should Nurse Kenneth document as the total intake for the 8 hour shift?
A. 850 ml B. 1300 ml C. 1350 ml D. 350
ml

ANSWER: A
RATIONALE:
The client has the following nursing order:
Gentamycin 80 mg IV piggy back in 50 ml D5 Water over 30 minutes
Ranitidine 50 mg IV in 50 ml D5 water piggy back in 30 minutes
Packed red blood cells (RBC) 250 ml to run for 3 hours
D5LR IV 1000 ml at 125 ml/hr. This will run for 8 hours

AMOUNT OF FLUIDS TIME NEEDED TO CONSUME

Gentamycin 80 mg in 50 ml D5W 30 minutes


Ranitidine 50 mg IV in 50 ml D5W 30 minutes
Packed red blood cells (RBC) 250 ml 3 hours
D5LR IV 1000 ml at 125 ml/hr 4 hours
TOTAL: 850ml TOTAL: 8 hours
NOTE: The D5LR should originally run for 8 hours. However, because of the other medication and
BT that is needed to transfused the total time that D5LR should be transfused is 4 hours.
Therefore, only 500 ml of D5LR is transfused. Wherein the other half would be endorsed to the
next shift
27. While reading Jonathans chart, you read the laboratory findings as:
-serum potassium 2.2 mEq/L -sodium 129 mEq/L -Calcium 7.5 mg/L
(N: 3.5-5 meq ) (N: 135-145 mg ) (N: 8.2-10.2 mg )
The nurse would anticipate / prepare which of the following IV solutions to be prescribed?
A. Sodium Chloride 0.45 % C. Dextrose 5% in Lactated Ringers
solution
B. Dextrose 5% in water D. Normosol
ANSWER: C
RATIONALE:
The client in the situation is experiencing hyponatremia, hypokalemia and hypocalcemia. The solution
of choice should be the one that contain these three electrolytes. Dextrose 5% in Lacteted Ringes
solution contains sodium (130 mEq/L), potassium (4 mEq/L), chloride (109 mEq/L) and calcium (3
mEq/L).
OPTION A is INCORRECT. Sodium chloride 0.45% only provides sodium and chloride.
OPTION B is INCORRECT. Dextrose 5% in water contain no electrolyte.
OPTION D is INCORRECT. Normosol is used for electrolye replacement solution. It contains sodium,
potassium and chloride.
REFERENCE: Smeltzer, Suzanne C. Brunner and Suddarths Medical-Surgical Nursing 12th Edition,
(2010), Volume 1, p. 272.

28. Nurse Kenneth continued to monitor Jonathan who has an ongoing IV and packed red blood cells
(PRBC) transfusion. The client complains of headache, backache and the temperature began to spike.
Rank the action of the nurse according to priority:
1. Refer to the attending physician 4. Keep the vein open with
NSS
2. Assess the client 5. Document observation and
intervention
3. Close the roller clamp of the PRBC
A. 2, 3, 4, 1, 5 B. 3, 2, 1, 4, 5 C. 3, 4, 2, 1, 5 D. 1,
2, 3, 4, 5

ANSWER: C
RATIONALE:
The client in the situation is experiencing acute hemolytic reaction, the most serious complication of
blood transfusion. This complication occurs when the donor blood is incompatible with that of the
recipient. Diligent care should be carried out on a client experiencing complication. The following action
should be done:
The nurse should first discontinue the transfusion immediately after recognizing the problem (3).
NOTE: When the transfusion is discontinued, the blood tubing must be removed as well. Uses new
tubing for the normal saline infusion.
The nurse then maintain vascular access with normal saline (2)
The nurse should further assess the client for manifestations before reporting to the clients attending
physician
The nurse then notify the clients primary health care provider immediately (1)
After which, the nurse should document his/her course of action (5)
OPTION A, B and D are INCORRECT. Refer to the explanation above.
REFERENCE: Kozier and Erbs Fundamental of Nursing, 8th Edition, (2007), Volume 2, p. 1474.

29. Nurse Kenneth identifies risk for wound complications. In case of wound evisceration, the IMMEDIATE
action of the nurse is to:
A. Instruct the client to stay quiet in bed as you call for help
B. Apply clean abdominal binder and place the pillow on top of the wound
C. Cover the wound with sterile gauze wet with sterile NSS
D. Call for the surgeon stat
ANSWER: C
RATIONALE:
Evisceration (protrusion of wound contents), when this complication happens, the nurse should
covered the protruding wound content using a sterile dressing moistened with sterile saline solution.
OPTION A is INCORRECT. This intervention is accurate, but it should be done after the nurse cover
the wound.
OPTION B is INCORRECT. This intervention would contaminate the wound and would result to
infection.
OPTION D is INCORRECT. After the nurse covered the wound and instructed the client to remain still,
the nurse may call to surgeon to surgically repair the evisceration.
REFERENCE: Smeltzer, Suzanne C. Brunner and Suddarths Medical-Surgical Nursing 12th Edition,
(2010), Volume 1, p. 480.

30. Jonathan has been NPO since he was operated and asks the nurse when he can have food. Nurse
Kenneths most appropriate response is:
A. The dietitian will make their rounds in a while to assess you and other postoperative clients.
B. The surgeon will make their rounds to assess your readiness to take in your preferred diet.
C. Clear soup will be served as soon as you have bowel sounds
D. You can have sips of water for the mean time

ANSWER: C
RATIONALE:
Taking food by mouth stimulates digestive juices and promotes gastric function and intestinal
peristalsis. The return to normal dietary intake should proceed at a pace set by the client. The nature of
the surgery and the type of anesthesia. Once the client have bowel sound, clear liquid (Clear soup,
water, juice and tea) may be given. As soon as the client tolerated soft foods well, solid food may be
given.
OPTION A is INCORRECT. This nursing statement does not answer the clients concern directly
OPTION B is INCORRECT. Again, this nursing statement does not answer the clients concern directly.
OPTION D is INCORRECT. The client is NPO, meaning, he cannot anything by mouth. Therefore, this
intervention is incorrect.
REFERENCE: Smeltzer, Suzanne C. Brunner and Suddarths Medical-Surgical Nursing 12th Edition,
(2010), Volume 1, p. 476.

SITUATION: Nelly, a 20-year-old college student, was just diagnosed of psoriasis. Her self-esteem is
adversely affected because of the skin lesions and painful joints that she has been having especially when
she is stressed or when she is exposed to extreme environmental temperature
31. Although the primary cause is unknown, a combination of genetic makeup and environmental stimuli
may trigger the onset of disease. Nurse Rad is correct when he explains to Nell the nature of psoriasis
as:
A. Chronic, infectious B. Acute, noninfectious C. Chronic, noninfectious
D. Acute, infectious

ANSWER: C
RATIONALE:
Psoriasis is one of the most common skin disorders. This skin disorder is often chronic and not
infectious or contagious

32. Nurse Rad needs no further study about psoriasis when he says that the lesion of this disease is:
A. Red raises patches, covered with silvery scales
B. Purple raised patches, covered with pus-filled red scales
C. Red flat patches, covered with silvery scales
D. Purple, flat patches, covered with pus-filled red scales

ANSWER: ARATIONALE:
33. Topical applied agents are used to slow the overactive epidermis without affecting other tissues.
Medications include tar preparation, anthralin, salicylic acid and corticosteroid. To increase the
effectiveness of the corticosteroid, Nurse Rad should?
A. Apply it on dry skin after bathing C. Apply it on moist skin after bathing
B. Apply occlusive dressing D. Apply it using sterile gloves

ANSWER: B
RATIONALE:

34. The goals of medical management for Nellys condition are the following except:
A. To slow the rapid turnover of epidermis C. To promote resolution of the
psoriatic lesions
B. To temporarily cure this disease D. To promote the natural cycles of the
disease

ANSWER: B
RATIONALE:
There is no known cure for psoriais
35. Another patient consulted because of a band-like pattern of painful vesicular lesions on her anterior
thigh. Physicians diagnosis is Herpes Zoster. Which of the following data in the patients history will
Ginny consider as a precipitating factor?
A. High protein diet C. Smokes 2 cigarettes a day
B. Hypersensitivity reaction to seafoods D. Post radiation therapy for typhoid
problem

ANSWER: D
RATIONALE:

SITUATION: A 69 year old female patient was admitted in the Emergency Department (ED) via an
ambulance from a nearby restaurant. Patient was drowsy and was unable to identify herself. Blood
pressure was 150/90, tachycardic and respiration shallow with a rate of 30/minute. Companion of the
patient claimed that while laughing, the patient got choked with a piece of pork meat and was unable to
breathe for quite some time until somebody successfully got it out of her throat.
36. Physicians admitting diagnosis is acute respiratory acidosis. If you were the ED nurse who admitted
the client, which of the following will you expect?
A. A normal pH and a PaCO2 greater than 45 mm Hg
B. A pH less than 7.35 and a PaCO2 greater than 45 mm Hg
C. A pH more than 7.35 and a PaCO2 lower than 45 mm Hg
D. A pH less than 7.35 and a PaCO2 of 45 mm Hg

ANSWER: B
RATIONALE:

37. The initial arterial blood gases results of the client revealed a normal bicarbonate level. Which of the
following statements will the nurse accept as TRUE that would explain the specific finding?
A. Serum bicarbonate will remain unchanged in all types of respiratory acidosis
B. The kidneys can modify only the excretion rate of acids
C. The compensatory response of the kidneys occurs only over hours to days
D. A change in serum bicarbonate is noted only in cases of respiratory acidosis

ANSWER: C
RATIONALE:
This statement is accurate. Compensation only occurs over hours to days. The kidney will try to
compensate by excreting hydrogen ion in the urine in exchange for bicarbonate ions.
OPTION A is INCORRECT. The level of bicarbonate is highly variable.
OPTION B is INCORRECT. The kidney can modify both the excretion of acid and bicarbonate in the
body.
OPTION D is INCORRECT. Respiratory alkalosis also exhibits change in bicarbonate level. Therefore,
this statement is inaccurate.
REFERENCE: Smeltzer, Suzanne C. Brunner and Suddarths Medical-Surgical Nursing 12th Edition,
(2010), Volume 1, p. 297.

38. You understand that the alteration in the mental status of the client is primarily due to the acute effects
of which of the following?
A. Hyperventilation B. Hypercapnia ` C. Tachypnea
D. Hypoxemia

ANSWER: B
RATIONALE:

39. Related to temporary airway obstruction, the admitting nurse identified, Impaired Gas Exchange as a
priority nursing diagnosis. Which of the following interventions will you consider to be the MOST
appropriate to determine if normal gas exchange has been regained?
A. Administer prescribed bronchodilator ` C. Monitor arterial blood gases redrawn
every 2 hours
B. Maintain on oxygen inhalation as ordered D. Place on Fowlers position as tolerated

ANSWER: C
RATIONALE:
The question is pertaining to evaluation of a client with a nursing diagnosis of Impaired Gas Exchange
Reassessment would help the nurse to find cues whether the client condition is getting better or getting
worse.
OPTIONS A, B and D are INCORRECT. These are nursing intervention that are usually carried out to
help the client with respiratory acidosis.
40. The client is closely observed for signs of dysrrhythmia. If you were the nurse at the bedside, which of
the following waves in the cardiac monitor will you consider as the wave that represents the contraction
of the ventricles?
A. QT interval B. QRS complex C. PR interval
D. ST segment

ANSWER: B
RATIONALE:
Clients with respiratory acidosis usually have increased in blood hydrogen ion level. This will lead to
hyperkalemia and if the condition worsens dysrhythmias may result. The wave that represents
ventricular contraction is the QRS complex.
OPTION A is INCORRECT. The QT interval represents the total time required for ventricular
depolarization and repolarization. The QT interval is measured fro mthe beginning of the QRS complex
to the end of the T wave. Prolonged Qt interval may lead to a unique type o ventricular tachycardia
called torsades de pointes.
OPTION C is INCORRECT. The PR interval represents the time required for atrial depolarization as
well as the impulse delay in the AV node and the travel time to the purkinjie fiver. It normally measures
from 0.12 to 0.20 seconds (five blocks)
OPTION D is INCORRECT. ST segment represent early ventricular repolarization.
REFERENCE: Ignatavicius and Workmans Medical-Surgical Nursing: Critical Thinking for Collaborative
Care 5th Edition, (2006), Volume 1, pp. 285, 713-714

SITUATION: As an ED Nurse you should always be ready for any kind of emergency situation.
41. Noel, a 30 year old factory worker is brought to the emergency department with multiple lacerations
and tissue avulsion of the right hand after catching the hand in a product conveyor belt. When
asked about tetanus immunization, he says Ive never had any vaccinations. You will anticipate
administration of:
A. Tetanus toxoid
B. Immunoglobulin and tetanus-diphteria toxoid
C. Immunoglobulin
D. Immunoglobulin, tetanus diphtheria toxoid, and pertussis vaccine

ANSWER: D
RATIONALE:
As stated by the client, since he never had any vaccinations, immunoglobulin, tetanus diphtheria
toxoid, and pertusis vaccine should be given.

42. Millard is brought to the ED by his co-worker after suffering from amputation of a left middle finger. As
his nurse, which of the following should you avoid to do?
A. Placing the wrapped finger in a plastic bag
B. Cleansing the stump area with normal saline
C. Wrapping the amputated finger in sterile gauze moistened with saline
D. Placing the amputated finger directly on ice

ANSWER: D
RATIONALE:
Injuries to the fingertips are common in accidents at home work, and play. They can occur when a
fingertip slams in a car door, while chopping vegetables or even when clearing debris form a
lawnmover of snowblower. Fingertip injuries can be crushing, tearing, or amputating injuries to the tops
of finger and thumbs. Fingertips are rich with nerve and are extremely sensitive. Without prompt and
proper care, a fingertip injury can distrupt complex function of the hand, possibly resulting in permanent
deformity and disability
FIRST AID
When preparing to see a doctor:
Elevate the injury and apply ice to reduce bleeding and swelling.
Cover the fingertip wound with a dry sterile dressing
Immobilize the affected hand and wirst with a short splint
If the fingertip is completely cut off:
Gently clean the amputated part with normal saline solution
Cover it in gauze wrap
Put it in a watertight bad
Place the bag on ice
DO NOT PUT THE AMPUTATED PART DIRECTLY IN ICE BECAUSE FURHTER DAMAGE
MAY RESULT
NOTE: Priority care to amputated part is to keep it viable in case reattachment efforts are warranted.
OPTIONS A, C and D are INCORRECT. Please refer to the explanation above.
43. Justine, a 7 year old child is brought to the ED by his mother complaining of arm pain after falling off a
swing at school. After assessing Justines pain as 8 on a scale of 1-10, what will be your priority
activity?
A. Assume that Justine is too young to verbalize how much pain he is experiencing
B. Obtain further information about his pain using age-appropriate tool.
C. Give Justine a narcotic pain medication
D. Ask his mother to wait in the waiting room to find out if Justine gets better without his mother

ANSWER: C
RATIONALE:
As early as age three (3), they can accurately report the location and intensity of their pain. Therefore,
the nurses priority action in this situation is to administer pain medication.
OPTION A is INCORRECT. This intervention is not appropriate.
OPTION B is INCORRECT. This intervention may be done, however as stated earlier, if properly
assessed, this client can appropriately report pain.
OPTION D is INCORRECT. This statement is inappropriate as this may further cause the client to cry.
REFERENCE: Koziers Fundamentals of Nursing 8th Edition, (2008), Volume 2, p. 1194.

44. During the primary assessment of Laurence, who has sustained multiple trauma, you observe that his
right pedal pulses are absent and the leg is swollen. Your priority action is to:
A. Initiate isotonic fluid infusion through two large bore IV lines
B. Finish the airway, breathing, circulation, disability survey
C. Send the blood to the laboratory for a complete blood count (CBC)
D. Assess further for a cause of the decreased circulation

ANSWER: B
RATIONALE:
The nurses priority action at this time is to finish the primary survey which consist of assessment of
airway, breathing, circulation and disability.
OPTIONS A, C and D are INCORRECT. These intervention should be done after the primary survey to
the client.

45. Anton, 20 years old college student, unconscious, is admitted to the ED about 45 minutes after
ingesting approximately 30 diazepam (Valium) tablets. The physician prescribes gastric lavage. As
a nurse, your first action when implementing the order is to:
A. Assist the physician to intubate Anton C. Insert a large-bore nasogastric tube
B. Position Anton on his side D. Prepare a 50 ml syringe with saline

ANSWER: B
RATIONALE:
Gastric lavage is the aspiration of stomach contents and washing out of the stomach by means of a
large-bore gastric tube. Gastric lavage is contraindicated after acid or alkali ingestion, in the
presence of seizures, or after ingestion of hydrocarbons or petroleum distillates.
The following steps should be taken into consideration when doing gastric lavage:
1. Remove dentures and inspect the oral cavity for loose teeth. This helps prevent aspiration and
airway obstruction
2. Measure the distance between the bridge of the nose and the xiphoid process.
3. Lubricate the tube with water soluble lubricant. This intervention helps the entry of the tube and to
prevent trauma
4. Place the client in a left lateral position with head lowered about 15 degrees
5. Pass the tube orally while keeping the clients head in a neutral position. Pas the tube to the
adhesive marking or about 50cm. If the client is conscious, instruct him to swallow to assist with the
passage of the tube
6. Aspirate the stomach content with the syringe attached to the tube before instilling water or an
antidote.
7. Remove the syringe. Attach the funnel to the end of the tube or use a 50 mL syringe to instill
solution in the gastric tube
8. Elevate the funnel above the clients head and pout 150 mL to 200 mL solution into the funnel
9. Save sample of the first two washing
10. Repeat the lavage procedure until the returns are relatively clear and no particulate matter is seen
OPTIONS A, C and D are INCORRECT. Please refer to the explanation above.
REFERENCE: Smeltzer, Suzanne C. Brunner and Suddarths Medical-Surgical Nursing 12th Edition,
(2010), Volume 1, p. 2176.

SITUATION: Nurse Leonora is conducting health teaching to a group of clients who are worried about
cancer.
46. Nurse Leonora assessing oral cancer risk factors in a client with a persistent sore on his tongue asks
about:
A. Consumption of highly spiced foods C. Regular use of dental floss
B. Thumb sucking or pacifier as a child D. Tobacco use in any form

ANSWER: D
RATIONALE:
The etiology of oral cancer is multifactorial, with the published research suggesting that exposure to
tobacco and alcohol coupled with genetic predisposition are major contributors to the disease. The
combination of smoking and alcohol seems to present the greatest risk for clients. HPV-16 and 18 has
been identified to be related to the development of oral cancer
OPTIONS A, B and C are INCORRECT. These risk factors are not closely related to the development
of oral cancer

47. In communicating bad news to a client newly diagnosed with cancer, the role of the nurse is to:
A. Facilitate C. Provide privacy
B. Get advance directives D. Discuss end-of-life issues

ANSWER: C

48. A 50-year-old woman who is nulliparous is at risk for what type of cancer?
A. Cervical B. Uterine C. Breast
D. Ovarian

ANSWER: B
RATIONALE:

OPTION A is INCORRECT.
OPTION C is INCORRECT.

OPTION D is INCORRECT. The incidence of this type of cancer increases after 40 years of age and
peaks in the early 80s; the median age of affected women is 63 years. The frequency of ovarian cancer
is highest in industrialized countries, except for Japan, where it is low. The incidence seems to be
remaining constant, and 5-year survival has improved (Bhoola & Hoskins, 2006). Pregnancy and use of
oral contraceptives decrease risk. Mutations of BRCA1 and BRCA2 increase risk; the lifetime risk for
women with these mutations is 28% to 40% (the higher percentage is in Ashkenazi Jews). A woman
with ovarian cancer has a threefold to fourfold increased risk of breast cancer, and a woman with
breast cancer has an increased risk of ovarian cancer. A family history, older age, low parity, and
obesity may increase risk of ovarian cancer. However, most women who develop ovarian cancer have
no known risk factors, and no definitive causative factors have been determined.
49. Clients who have history of irritable bowel syndrome area at risk for what type of cancer?
A. Bladder cancer B. Colon cancer C. Pancreatic cancer
D. Renal cancer

ANSWER: B
RATIONALE:

OPTION A is INCORRECT.

OPTION C is INCORRECT.
50. A 50-year-old man with a solid tumor on his neck asks about the meaning of stage T0N0M0. The nurses
best reply is:
A. The lesion is benign and not cancerous C. Another test should be done
to confirm the lesion
B. There is no evidence of primary tumor D. Have another pathologist
see the result

ANSWER: B
RATIONALE:

OPTION A is INCORRECT. To say if the tumor is benign or cancerous, biopsy should be used
OPTIONS C and D are INCORRECT. These actions are not necessary

SITUATION: Nurse Marie is assigned in the Oncology Unit handling clients with breast cancer
51. Which of the following is not a predisposing factor of Breast Cancer?
A. Menarche at 13 years old C. Late menopause
B. Height of 53 and weight of 160 lbs D. Family history

ANSWER: A
RATIONALE:
Early menarche is risk factor of breast cancer. Onset of menstrual period at age of 13 is not early
menarche.

52. The clients relative asked Nurse Marie how often she should perform BSE. The nurse s Best response
would be:
A. Every month 5 to 7 days after menses start. C. Every month in any date.
B. Every month 5 to 7 days before menses start. D. Whenever the client is
convenient.

ANSWER: A
RATIONALE:

53. The client has been scheduled to undergo modified radical mastectomy (MRM). The client asks the
nurse about the procedure. The nurse replied to the client that the this procedure involves removal of:
A. Entire breast tissue, including the nipple-areola complex and a portion of the axillary lymph
nodes
B. Entire breast tissue, including the nipple-areola complex, a portion of the axillary lymph
nodes and pectoralis muscle
C. Entire breast tissue and the nipple-areola complex
D. The mass only
ANSWER: A
RATIONALE:

OPTION B is INCORRECT. if the breast tissue, nipple-areola complex, lympnodes and muscles are
removed, this procedure is known as radical mastectomy

54. The physician has ordered Fluorouracil 700 mg Iv once a week to a 60-year-old client with malignant
tumor of the breast. When the client heard this, she says to the nurse, Am I going to lose my hair?
Which is the best response by the nurse?
A. Hair loss is normal and you can use a wig until you hair grows back.
B. Fluorouracil usually does not cause you to lose your hair.
C. The physician will prescribe a medication to prevent this side effect from occurring.
D. Losing your hair is less traumatic than losing your breast.

ANSWER: A
RATIONALE:
Chemotherapeutic agents affect both cancerous and healthy rapidly dividing cells. One of the
commonly affected is the hair. Clients may experience alopecia and commonly it occurs 2-3 weeks
after the initial therapy. This is a normal side effect of chemotherapy. With this, client may be instructed
to wear wig or baseball cap. Usually re-growth occurs at about 8 weeks after the last therapy
55. A 30-year-old female client who was discharged following a breast conserving surgery, axillary node
dissection and radiation therapy calls the clinic to report her arm is red, warm to touch, and slightly
swollen. Which of the following actions should the nurse suggest?
A. See the your attending physician immediately C. Schedule an appointment
within 2 to 3 weeks
B. Apply warm compresses to the affected area D. Elevate the arm on two
pillows

ANSWER: D
RATIONALE:
The client reports to the nurse the following findings: affected arm is red, warm to touch and slightly
swollen. These manifestations may be indicative of lymphedema which is a common complication of
breast conserving surgery for breast cancer. This complication happens if the affected arm is used to
lift heavy objects. To relieve lymphedema, the nurse should instruct the client to elevate the arm above
the level of the heart by using two pillows

3
NP4 (1-70)
SITUATION: Mr. Sta. Rita, a post acute myocardial infarction (AMI) on his 2nd day post attack is assigned
to you, the physician said his recovery is uneventful. The following questions apply.
1. Morphine sulfate intravenous (I.V.) was prescribed for pain. The nurse understands that morphine
sulfate I.V. was preferred because of two reasons. These are:
1. Bypasses the variable rates of absorption 3. Elevates enzyme levels
2. Increases cardiac output 4. Rapid onset of action
A. 2 and 3 B. 2 and 1 C. 3 and 4 D. 1
and 4

ANSWER: D
RATIONALE:
Morphine sulfate is used in relief of moderate to severe acute and chronic pain; relief of pain of
myocardial infarction; relief of dyspnea of acute left ventricular failure and pulmonary edema;
preanesthetic medication.
Morphine sulfate actions is that it binds to opiate receptors in the CNS, causing inhibiting of ascending
pain pathways, altering the perception of and response to pain; produces generalized CNS depression.
In myocardial infarction morphine sulfate IV is given primary to decreases pain. In addition to that,
this medication also decreases the circulating cathecolamines thereby decreasing the preload, and
afterload and subsequently the heart rate and blood pressure which has a direct effect in the oxygen
demand of the heart. IV morphine sulfate was the drug of choice simply because of two reasons, since
this medication is given through Iv it bypasses the variable rate of absorption and the onset of
action upon administration is usually 5 10 minutes
OPTIONS A, B and C are INCORRECT. Statements on No. 2 and 3 are inaccurate reagarding the
information related to the use of IV morphine sulfate.
As stated earlier, this medication decreases the circulating catecolamine, thereby decreasing the
preload and the after. If these components are decrease, cardiac output also decreases. These
conditions occur to balance the myocardial oxygen demand.
Morphine sulfate does not elevate enzymes evel, not decreases the enzymes levels that are related to
myocardial infarction.

2. Mr. Sta. Rita is taking Aspirin, a platelet inhibitor. The client understood the nurses instruction on
how to take the drug if he:
A. Swallowed medicine in small amount of water C. Chewed and allowed the drug to
dissolve with saliva
B. Took the medicine two hours before meals D. Took the medicine with meals

ANSWER: D
RATIONALE:
Aspirin is an antiplatelet medication that is used in clients with myocardial infarction. This medication
prevents platelet aggregation. The major side effects that can occur during the intake of this medication
is gastrointestinal upset/ulceration. This can lead to bleeding since this medication inhibits platelet
aggregation. Therefore, it is necessary to instruct the client to take the medication with food.
OPTION A is INCORRECT. This intervention would not prevent the occurrence of gastrointestinal
upset. Besides, the medication should be taken with a FULL glass of water.
OPTION B is INCORRECT. This intervention should be avoided as this may lead to gastrointestinal
upset.
OPTION C is INCORRECT. The nurse should instruct the client to dont chew the medication.

3. The clients wife observes the facial expression of Mr. Sta. Rita and interprets that her husband is
in pain. She asks the nurse, What is causing the pain? The nurse responded that:
A. Release of tissue substances during inflammatory process can stimulate pain receptors.
B. Pain is felt when the myocardial muscles contract rapidly
C. Pain is triggered by the high blood pressure
D. Chest pain occurs when the oxygen demand of the heart is not met

ANSWER: D
RATIONALE:
The main problem in myocardial infarction is the imbalance between the oxygen demand and supply of
the heart. This imbalance will result to the shifting of metabolism from aerobic to anaerobic metabolism
and can lead to the production of LACTIC ACID. The nerve endings in the heart is sensitive with lactic
acid, and this can cause chest pain.
OPTION A is INCORRECT. This statement is somewhat correct, but it does not answer the concern of
the wife directly
OPTION B is INCORRECT. Ventricular tachycardia may occur in clients with myocardial infarction but
it does not cause the pain.
OPTION C is INCORRECT. Increase blood pressure happens with pain, but it does not directly cause
the pain.
REFERENCE: Smeltzer, Suzanne C. Brunner and Suddarths Medical-Surgical Nursing 12th Edition,
(2010), Volume 2, p. 774.

4. One of the priority nursing diagnoses is Ineffective Tissue Perfusion. Which of the following would
you watch for as the first indication of altered perfusion?
A. Adventitious lung sounds C. Change in the level of
consciousness
B. Presence of dysrhythmias D. Abnormal heart sounds

ANSWER: B
RATIONALE:
Ineffective Tissue Perfusion pertains to altered blood flow to myocardial tissue. In MI. decrease
myocardial contractility and ventricular compliance caused by necorsis may lead to dysrhythmias. If
dysrhythmia occurs, cardiac output and blood pressure may be decreased and can lead to ineffective
tissue perfusion to various organ in the body.
OPTIONS A and D are INCORRECT. Adventitious lung sounds and abnormal heart sounds may
indicate left ventricular failure.
OPTION C is INCORRECT. Change in the level of consciousness is the earliest manifestation if we are
talking about ineffective tissue perfusion on the brain or CNS

5. During episodes of chest pain, which of the following procedures would the nurse expect to be
prescribed to provide assessment for myocardial infarction?
A. Electrocardiography B. Echocardiography C. Radionuclide imaging
D. Angiography
ANSWER: B
RATIONALE:
Ineffective Tissue Perfusion pertains to altered blood flow to myocardial tissue. In MI. decrease
myocardial contractility and ventricular compliance caused by necorsis may lead to dysrhythmias. If
dysrhythmia occurs, cardiac output and blood pressure may be decreased and can lead to ineffective
tissue perfusion to various organ in the body.
OPTIONS A and D are INCORRECT. Adventitious lung sounds and abnormal heart sounds may
indicate left ventricular failure.
OPTION C is INCORRECT. Change in the level of consciousness may also occur but in the later
stage.
REFERENCES: Ignatavicius and Workmans Medical-Surgical Nursing: Critical Thinking for
Collaborative Care 5th Edition, (2006), Volume 1, p. 853.
Smeltzer, Suzanne C. Brunner and Suddarths Medical-Surgical Nursing 12th Edition, (2010), Volume
2, pp. 775-776.

SITUATION: The nurse is working in the Intensive Care Unit during the night shift when a code is called for
ventricular fibrillation. The nurse is assisting at a code and the physician is going to defibrillate the client.
6. Of the following items, which is the only one that the nurse does not need to remove from the client
just before the client is defibrillated?
A. Back board C. Nitroglycerin patch
B. Oxygen D. Pulse oximetry machine plugged into an
electrical socket

ANSWER: A
RATIONALE:
Backboard during defibrillation would not cause any safety issues. In fact, this item is necessary to
stabilize the clients back
OPTIONS B, C and D are INCORRECT. Oxygen, nitroglycerin patch and pulse oximetry plugged into
an electrical socket may cause fire if not removed when a client is defibrillating

7. When defibrillating a client, the paddles are placed anterolaterally. One of the paddles is positioned
to the right of the upper sternum. Where would the nurse place the other paddle?
A. Over the 5th or 6th intercostal space at the left anterior axillary line
B. Directly over the heart at the precordium to the left of the lower sternal border
C. Under the clients body beneath the heart and immediately below the scapula
D. On the right side of the chest on the same level as the nipple line

ANSWER: A
RATIONALE:
8. After the placement of the paddles for defibrillation, the nurse then:
A. Orders any personnel away from the client, charges the machine, and defibrillates through the
console
B. Performs cardiopulmonary resuscitation for 1 minute before defibrillation
C. Charges the machine and immediately pushes the discharge buttons on the console
D. Administers rescue breathing during the defibrillation

ANSWER: A
RATIONALE:

9. When defibrillation is used:


A. Between 20 and 25 lb of pressure should be exerted on each paddle
B. The defibrillator should discharge at 100 W/sec per kg of body weight
C. The discharge shock needs to be timed to the T wave
D. All of the above are necessary

ANSWER: A
RATIONALE:

OPTIONS B, C and D are INCORRECT. The defibrillator is discharged at the rate of 200 J initially, not
watt. In addition to that, defib is not synchronized, meaning, the health care provider can discharge
shock at any wave.

10. The nurse observes that the attempts to convert the ventricular fibrillation were unsuccessful.
Based on an evaluation of the situation, the nurse determines that which action would be best?
A. Performing CPR for 1 minute, the defibrillating up to three or more times at 360 joules
B. Performing CPR for 5 minutes, then defibrillating three or more times at 400 joules
C. Preparing for the administration of sodium bicarbonate intravenously
D. Terminating the resuscitation effort

ANSWER: A
RATIONALE:

SITUATION: Nurse Lito is assigned to take care for clients with different diseases. Having the basic
knowledge about these diseases is important to provide quality care and prevent further damage or
complication
11. Which of the following factors has increased the clients risk for developing Buergers disease?
a. History of cigarette smoking C. Age and gender

b. Occupational exposure to radiation D. History of hypertension

ANSWER: A
RATIONALE:

12. When assessing an individual with peripheral vascular disease, which of the following
manifestation would indicate complete arterial obstruction in her lower leg?
a. Aching pain in the left calf C. Numbness and tingling in
the left leg
b. Burning pain in the left calf D. Coldness of the left foot and
ankle

ANSWER: D
RATIONALE:
Peripheral vascular disease (PVD) Includes disorders that alter the natural flow of blood through
the arteries and veins of the peripheral circulation.
It affects the upper and lower extremities (most commonly affected)
Atherosclerosis is the most common cause of PVD
Fatty substances accumulate at the site of vessel wall injury and alter or totally obstruct /occlude
the blood flow within the arteries
Physical assessment includes the following: Depending on the severity
Intermittent claudication (client walks a certain distance before cramping, burning,
or pain forces them to stop. The pain subsides after rest. If the clients resume
walking a certain distance pain occurs again) This is the reason why clients with PVD
seek medical care
Loss of hair on the affected extremity (calf, ankle and foot)
Dry, scaly, dusky, pale or mottled skin and thickened toe nails
With severe arterial disease (complete obstruction), the extremity is cold and gray-blue
(cyanotic)
OPTIONS A and B are INCORRECT. These manifestations are more related to deep vein thrombosis
OPTION C is INCORRECT. Numbness and tingling may be due to damage to the nerve (more
common) or decrease blood supply. In addition, occurrence of numbness and tingling indicates
decrease bloof flow, not complete obstruction of the blood flow
13. Nurse Lito is caring for Rosita diagnosed with acute arterial occlusion of the left lower extremity.
Which of the following is important for the nurse to observe to prevent further tissue damage?
a. Blood pressure and heart rate changes C. Metabolic acidosis
b. Gradual or acute loss of sensory and motor function D. Swelling in the left lower extremity

ANSWER: B
RATIONALE:
14. Which nursing action would be most effective in preventing venous stasis in the immobilized client?

a. Raise the foot of the bed for 1 hours, then lower it down
b. Vigorous massage of the lower extremities every 6 hours
c. Active range of motion of the upper body
d. Assist the client to walk as soon as and as often as possible

ANSWER: D
RATIONALE:
15. Nurse Lito is assisting a student nurse in giving instruction to a client about to wear anti-embolic
stocking. Which of teaching made by the student needs no reinforcement by nurse Lito?
a. Wear the stockings until the physician tells you to remove them.
b. Even when you sleep, you should wear your stockings.
c. You should remove the stockings every four hours for 30 minutes.
d. Before getting out of bed in the morning, you should put on the stocking.

ANSWER: C
RATIONALE:

OPTIONS A, B, and D are INCORRECT.


Nursing interventions for anti-embolic stocking
Wear the stocking as prescribed, usually during the day and evening (even when sleeping)
Put the stockings on upon awakening and before getting out of bed
When applying the stockings, do not bunch up and apply like socks. Instead, place your hand inside
the stockings and pull out the heel. Then place the foot of the stocking over your foot and slid the
rest of the stockings up.
Do not push stocking down for comfort, because they may function like a tourniquet and further
impair venous return. Put on a clean pair of stockings each day. Wash them by hand in a gentle
detergent warm water
If the stockings seems to be stretched out, replace them with a new pair

SITUATION: Nurses attitudes toward the pain influence the way they perceive and interact with clients in
pain.
16. Nurses should be aware that older adults are at risk of underrated pain. Nursing assessment and
management of pin should address the following beliefs EXCEPT:
A. Older patients seldom tend to report pain than the younger ones
B. Pain is a sign of weakness
C. Older patients do not believe in analgesics, they are tolerant
D. Complaining of pain will lead to being labeled a bad patient

ANSWER: C
RATIONALE:
The question is looking for a negative statement regarding pain belief in older clients. The statement,
Older patients do not believe in analgesics, they are tolerant is a false statement. Older clients still
believe in analgesic, however they often not report pain which is the reason they are not given
analgesic for

17. Nurses should understand that when a client responds favorably to a placebo, it is known as the
placebo effect. Placebos do not indicate whether or not a client has:
A. Conscience B. Disease C. Real pain
D. Drug tolerance

ANSWER: C
RATIONALE:
A positive response to a placebo dose is not indicative of a lack of real pain but only of the reality of the
placebo effect, which can be expected in 30% or more of any population. Because placebos fails to
relieve pain for many people it is recommended that the deceptive use of placebos be considered
unacceptable in the management of pain

18. You are the nurse in the pain clinic where you have client who has difficulty specifying the location
of pain. How can you assist such client?
A. The pain is vague
B. By charting-it hurts all over
C. Identify the absence and presence of pain
D. As the client to point to the painful are by just one finger

ANSWER: D
RATIONALE:
To ascertain the specific location of the pain, ask the individual to point the site of the discomfort. A
chart consisting of drawings of the body can assist in identifying pain location. The client marks the
location on the chart. This tool can be especially effective with clients who have more than one source
of pain

19. What symptom, more distressing than pain, should the nurse monitor when giving opioids
especially among elderly clients who are in pain?
A. Forgetfulness C. Constipation
B. Drowsiness D. Allergic reactions like pruritis

ANSWER: C

20. Physical dependence occurs in anyone who takes opiods over a period of time. What do you tell a
mother of a dependent when asked for advice?
A. Start another drug and slowly lessen the opioid dosage
B. Indulge in recreational outdoor activities
C. Isolate opioid dependent to a restful resort
D. Instruct slow tapering of the drug dosage and alleviate physical withdrawal symptoms

ANSWER: A
SITUATION: Some nurses often do not believe the clients complaint of pain. The primary role of the nurse
in pain management is to speak in behalf of the client. The following questions apply.
21. The nurse is caring for an arthritic client who was given Etoricoxile (Arcoxia) 80 mg for pain an hour
ago. The client stated that he is still experiencing pain. The nurse must:
A. Divert attention of client to a TV show C. Determine the clients pain score
B. Refer clients compliant to the physician D. Administer the next dose

ANSWER: C
RATIONALE:
In some cases, reason why medication has a little effect to pain is because of low dose administration.
But in the case of Arcoxia 80mg, this is enough dosage to relieve the pain. Determining the clients
severity of pain will allow the nurse to assess the intensity of pain before and after the medication was
administered. Then the nurse can now interpret if the medication is starting to effect since the
administration of Arcoxia is just an hour ago. If the pain of the patient is somehow decreasing, then the
medicine is just starting to produce its desired effect
OPTION A is INCORRECT. Though diverting of clients attention will somehow help, it will not totally
solve the pain
OPTION B is INCORRECT. Doing nursing intervention fist is important prior to referral of the case of
the client to the physician
OPTION D is INCORRECT. Administering the next dose is not an appropriate action since the
administered dose is high enough to address the pain of the client
22. A nurse is monitoring her postoperative clients. Nurses always asked their clients pain level after
taking the vital signs to:
A. Determine that the vital signs are not altered by pain
B. Follow the doctors orders
C. Ensure that pain assessment is done on a regular basis
D. Be consistent with the McCafferys guidelines on pain management

ANSWER: C
RATIONALE:
Pain management is considered such an important part of care that it is referred to as the fifth vital
sign to emphasize its significance and to increase the awareness among health care professionals of
the importance of effective pain management (American Pain Society, 2003). Identifying pain as the
fifth vital sign suggests that the assessment of pain should be as automatic as taking a patients
blood pressure and pulse. The Joint Commission (2005, p.1) standards state that pain is assessed
in all patients and that patients have the right to appropriate assessment and management of pain.
OPTION A is INCORRECT. Though pain might alter the normal levels of VS, this is not the primary
reason why pain is taken after VS monitoring
OPTION B is INCORRECT. Vital signs and pain monitoring is part of the role of the nurse which is
assessment

23. A client with cholecystitis complained of pain in the right shoulder and asks the nurse. Whats
wrong with me? Even my shoulder aches. Which is the BEST explanation of the nurse?
A. That is usually the case.
B. Sometimes pain from other organ is referred elsewhere in the body.
C. Would you like me to refer you to your doctor?
D. Does your other shoulder hurt too?

ANSWER: B
RATIONALE:
This statement gives explanation to the clients inquiry to the nurse. This type of pain is known as
radiating pain or referred pain
OPTIONS A, C and D are INCORRECT. Statements here are not a type of explanation to the query of
the client

24. The nurse is in-charge of a client on a long term Nitroglycerine sublingual tablets for angina
pectoris. What instruction of the nurse is APPROPRIATE for the client to maintain the efficacy of the drug?
A. Instruct your watcher to press the button every four hours.
B. Maintain a supply for duration of one year.
C. Replace sublingual tablets supply every three months.
D. Keep sublingual tablets in amber glass bottle.

ANSWER: D
RATIONALE:
NTG should be keep in an dark glass bottle because light or sun exposure may inactive the medication
OPTIONS B and C are INCORRECT. Nitroglycerin is volatile and is inactivated by heat, moisture, air,
light, and time. Patient should be instructed to renew the nitroglycerin supply every 6 months

25. After an abdominal surgery, which of the following instructions of the nurse is MOST appropriate
for a client who has a client-controlled analgesia device?
A. Instruct your watcher to press the button every four hours.
B. Try to tolerate the pain before your press the button.
C. Push the button when you feel the pain is starting.
D. Push the button every 30 minutes whether you feel pain or not.

ANSWER: C
RATIONALE:
The PCA pump permits the patient to self-administer continuous infusions of medication (basal rates)
safely and to administer extra medication (bolus doses) with episodes of increased pain or painful
activities. A patient experiencing pain can administer small amounts of medication directly into his or
her IV, subcutaneous, or epidural catheter by pressing a button. The pump then delivers a preset
amount of medication.
A PCA pump is electronically controlled by a timing device. The timer can be programmed to prevent
additional doses from being administered until a specified time period has elapsed (lock-out time) and
until the first dose has had time to exert its maximal effect. Even if the patient pushes the button
multiple times in rapid succession, no additional doses are released. If another dose is required at the
end of the delay period, the button must be pushed again to receive the dose
SITUATION: A nurse admitted a 20-year-old college student. Her chief complaints are fatigue, weakness
and sometimes dizziness. The patient is pale. The admitting diagnosis is iron deficiency anemia.

26. The nurse prepared the client for complete blood count (CBC) testing. The complete blood count
is normal if the result is:
1. Red blood cells 3.6 to 5.0 million/mm3 4. Hematocrit 37 to 47 %
2. Reticulocyte 1.0 % to 1.5 % of total RBC 5. Hemoglobin 6 to 9 g/dl
3. Hemoglobin 14-16.5 % g/dl 6. Hematocrit 40 % to 50 %
A. 1, 2, 3, and 6 B. 1, 3 ,5, and 6 C. 1, 2 ,3, and 4 D. 1,
2, 4, and 5

ANSWER: A

RATIONALE:

These laboratory values are within normal limit or level

OPTION A, B and C are INCORRECT. Hematorcit is below normal level (4), as well as the
hemoglobin (5)

27. After a thorough assessment and based on the laboratory findings, the diagnosis of iron
deficiency anemia is confirmed. The client asks the nurse what is the role of iron in the body? The
CORRECT response of the nurse is:
A. Iron prevents bleeding. C. The body cannot synthesize hemoglobin
without iron.
B. Iron gives the red color of our blood. D. Iron helps in the conduction of nutrients to
the body.

ANSWER: C

RATIONALE:

Iron is helps body to synthesize hemoglobin. .

OPTION A is INCORRECT. Vitamin K and platelet as well as clotting factor prevent bleeding

OPTION B is INCORRECT. Iron also gives color to the blood. However the most important function
of iron is that is helps the body to synthesize hemoglobin

OPTION D is INCORRECT. Iron does not help in the conduction of nutrients to the body instead it
helps in conduction of oxygen to the tissue of the body

28. Which of the following food enhance absorption of iron?


A. Cereals. C. Dairy products
B. Citrus fruits D. Green leafy vegetables.

ANSWER: B

RATIONALE:

Iron is best absorbed in an acidic environment, administering iron with citrus fruit which is acidic in
nature, increases its absorption
OPTION A, B and D are INCORRECT. Cereals, dairy products and green leafy vegetable do not
enhance absorption of iron. Instead these foods are high in vitamin B9

29. The client was prescribed Ferrous Sulfate as iron supplement. For better absorption, the nurse
would instruct the client to take this supplement:
A. With meals. B. 1 hour before C. After breakfast D. Before
going to bed.

ANSWER: B

RATIONALE:

Hcl production is increased if the client stomach is empty that why it is administered prior to meal for
better absorption

OPTION A, C and D are INCORRECT. Iron is best absorbed in an acidic environment.

30. Intramuscular supplementation of iron causes local pain and can cause stain in the skin. If you
are the nurse, which BEST technique of administration will you use?
A. Z track technique C. Vigorous rubbing of the injection site
after injection.
B. IV bolus. D. Intramuscular using deltoid

ANSWER: A

RATIONALE:

Administering IM preparation of iron using a Z track method prevents staining and pain by creating a
seal. This is done by pulling the skin to the side and administering the injection, after which skin is
released and when the skin returns to its normal position after the needle the needle is withdrawn, a
seal is formed over the intramuscular site thus creating a seal. In addition irritation of the site is less
causing decrease pain

OPTION B, C, and D are INCORRECT. These interventions could cause pain and staining of the skin

SITUATION: Caring for patients with immunologic problems or concerns can be very challenging and a
failed decision can be life threatening. Nurses should be prepared to handle such challenges. Knowledge
of how the immune system works is a basic responsibility.

31. You are preparing Hazel for discharge. She had spelenctomy following multiple injury sustained
from a vehicular accident. The MOST important precaution to teach Hazel regarding her immune function
is that she:
A. Will be at a decreased risk for developing allergies, so it will no longer be necessary for her to
worry about asthmatic allergies.
B. Should have yearly checkups because she is at high risk for cancer development now.
C. Will no longer develop a fever when she has an infection. She must learn to identify other
symptoms of infections.
D. Should avoid people with flu because it is hard for her to develop antibodies.

ANSWER: D
RATIONALE:
Adults who undergo splenectomy are already considered immunosuppresed therefore they should
avoid people with current infections.
OPTION A is INCORRECT. Splenectomy increases the risk of developing allergies
OPTION B is INCORRECT. Yearly checkups are advised not because of cancer development, but
because they are at risk of developing infections.
OPTION C is INCORRECT. They will still develop fever

32. Mrs. Jose, 40 years old tells you, My son has chickenpox. I am reluctant to visit him because I
am afraid to get herpes zoster. What will be your BEST response?
A. Herpes Zoster is caused by a virus different from the chickenpox virus.
B. If you already had herpes zoster, you cannot get them again.
C. Herpes Zoster is caused by the virus that is left over from when you had chickenpox.
D. If you already had chickenpox you are immune to herpes zoster.

ANSWER: C

RATIONALE:

Herpes zoster results from reactivation of varicella virus that has lain dormant in the cerebral ganglia
extramedullary ganglia of the cranial nerves) or in the ganglia of posterior nerve roots since a
previous episode of chickenpox

OPTION A is INCORRECT. Herpes zoster is caused by reactivation of varicella virus

OPTION D is INCORRECT. Having chickenpox doesnt give you immunity in developing herpes
zoster

33. Jane who is recovering from Hepatitis A asks you if she should take the vaccine to avoid
contacting the disease again. What is your BEST response?
A. No, your liver and immune system are too impaired at this time.
B. Yes, because Hepatitis A virus strain changes from year to year.
C. No, having had Hepatitis A has done the same thing a vaccination would.
D. Yes, because now you are more susceptible to have Hepatitis A.

ANSWER: C

RATIONALE:

Hepatitis A confers immunity against itself

OPTIONS A, B and D are INCORRECT. Hepatitis A are usually mild and develops immunity against
the disease

34. While you are preparing to administer a tetanus toxoid vaccination for Carlo who had a punctured
wound, he tells you that he had tetanus toxoid shot just one year ago. What will be your BEST response?
A. Antibody production slows down as you age. You need a booster.
B. You probably do not need another vaccination now.
C. It wont hurt to receive an extra dose of the toxoid.
D. You need this vaccination because the strain of tetanus changes every year.

ANSWER: A

RATIONALE:
Booster dose is needed in patient who took his tetanus toxoid vaccination year ago.

35. You are having a Mothers class for primigravidas who had recently given birth about immunity
that has been passed to the baby in utero. Which statement by the participant indicates that additional
teaching is needed?
A. I had chickenpox and I am immune to it, so my baby will not need to have chickenpox vaccine.
B. I had measles, so my baby will be protected against it until he is old enough to receive the MMR
vaccine.
C. Only certain antibodies were able to cross the placenta to protect my baby.
D. My baby received some antibodies from me before birth, and I will give him more when I
breastfeed.

ANSWER: A

RATIONALE:

Chickenpox vaccine is still needed even if the mother already had the disease. It will not give
protection to the baby

OPTIONS B, C and D are INCORRECT. They are correct statement which doesnt need further
teaching

SITUATION: Leif, a 56 year old farmer, was admitted in the Emergency Department (ED) because of
hematemesis accompanied by hematochezia. Leif is an alcoholic and is under treatment for cirrhosis of
the liver. His abdomen is enlarged and his lower extremities are edematous. Admitting physicians initial
diagnosis is ruptured esophageal varices.

36. Assessment reveals signs and symptoms of early compensatory hemorrhagic shock. If you were
the nurse who admitted Leif, which of the following will you consider as the compensatory mechanism
responsible for the increased heart rate and respiratory rate?
A. Stimulation of the sympathetic nervous system
B. Increase in size of the vascular bed due to peripheral vasodilation
C. Renin-angiotensin response
D. Release of adenocorticotrophic hormone from the hypothalamus

ANSWER: A

RATIONALE:

Stimulation of the sympathetic nervous system is the earliest compensatory mechanism that
occurring in response to hemorrhagic shock. This compensatory mechanism resulted in response to
tissue hypoxia. Stimulation of the adrenergic receptors causes increase in heart rate and blood
pressure from vasoconstriction. Increase respiratory rate is also observed.

OPTION C is INCORRECT. Stimulation of the renin-angiotensin response causes manifestations like,


cold clammy skin and decrease urinary output

OPTION D is INCORRECT. Release of adenocorticotrophic hormone occurs also as a compensatory


mechanism but does not directly cause increase in the heart rate and respiratory rate. Release of
adenocorticotrophic hormone affects the sodium level in the body, this causes increase reabsorption
of sodium and water.
37. To restore hemodynamic stability on the client, which of the following will the nurse expect to do
first?
A. Insertion of central arterial and venous catheters C. Endoscopic ligation of rupture
varices
B. Blood transfusion for blood replacement D. Administration of vasoactive and
inotropic drugs

ANSWER: A

RATIONALE:

Since the client is bleeding and experiencing shock, the foremost priority would be to restore and
maintain the fluid status of the client. Therefore, insertion of central arterial and venous catheters for
fluid replacement should be the priority nursing action.

OPTION B is INCORRECT. Transfussion is usually indicated once bleeding stops and the decision
depends on the amount of blood loss and the client laboratory work-up.

OPTION C is INCORRECT. Ligation of rupture varices is also important to halt bleeding, but the
priority is to restore the clients fluid volume.

OPTION D is INCORRECT. Administrations of vasoactive and inotropic drugs are also indicated but
again, the priority is to maintain fluid balance.

38. Hemodynamics measurement revealed stable vital signs and increased cardiac output. The
physician ordered treatment of the esophageal varices. Which of the following procedures will the nurse
expect to be done?
A. Upper endoscopy C. Exploratory Laparotomy
B. Intrahepatic portal systemic shunt D. Coagulation therapy

ANSWER: A

RATIONALE:

Endoscopic band ligation is usually indicated to treat esophageal varices. This procedure is done
using upper endoscopy.

OPTION B is INCORECT. Insertion of a tranjugular intrahepatic portal-systemic shunt is a


nonsurgical procedure performed in large interventional radiology department. This procedure is
indicated to control ascites and bleeding varices.

OPTIONS C and D are INCORRECT.

39. In the Intensive Care Unit, nursing orders required all nurses to assess regularly for early
manifestations of portal systemic encephalopathy. Which of the following will the nurse note during her
observation?
A. Presence of papilledema
B. Development of disorientation and incoherence
C. Occurrence of asterexis
D. Signs and symptoms of increased intracranial pressure

ANSWER: B
RATIONALE:

Disturbance or alteration in the clients level of consciousness is the earliest manifestation of portal
systemic encephalopathy.

OPTIONS A and D are INCORRECT. Manifestation of increase ICP usually occurs during the later
stage if the condition causes cerebral edema.

OPTION C is INCORRECT. Asterexis is observed during the later stage.

40. Serum ammonia level of the client remained to be elevated. The following may be considered by
the nurse to be true regarding this observation except:
A. Ammonia is formed as proteins and amino acids are broken down by intestinal bacteria
B. Ammonia accumulates in the blood due to inability of the kidney to excrete ammonia
C. Due to bleeding, a blood in the intestinal tract is digested as protein, thereby increasing serum
ammonia
D. Since liver function is destroyed, ammonia can no longer be converted to a less toxic form.

ANSWER: B
RATIONALE

SITUATION: Christopher, 55 years old, is in end-stage liver-failure. Past medical history reveals chronic
alcoholism, and a diagnosis of liver cirrhosis.

41. Which assessment finding indicates that the lactulose is effective in decreasing the ammonia
level in the client with hepatic encephalopathy?
A. Passage of two or three soft stools daily C. Daily deterioration in the client's
handwriting
B. Evidence of watery diarrhea D. Appearance of frothy, foul-smelling
stools

ANSWER: A

RATIONALE:
OPTION B is INCORRECT.

OPTION is INCORRECT. It is not humane to assess the hand writing of a client with asterexis

42. Which among the following interventions should the nurse NOT include when addressing hepatic
encephalopathy?
A. Assessing the client's neurologic status every 2 hours C. Evaluating the client's serum
ammonia level
B. Making sure the client's fingernails are short D. Monitoring the clients handwriting
daily

ANSWER: B

43. A nurse is assisting Christopher to fill out the dietary menu. The nurse advises the client to avoid
which of the following entree items that could aggravate the client's condition?
A. Fresh fruit plate B. Tomato soup C. Vegetable lasagna D.
Ground beef patty

ANSWER: D

RATIONALE:
44. Which outcome would be most appropriate for the client with hepatic coma?
A. The client is oriented to time, place and person C. The client increases oral intake to
2000 calories/day
B. The client exhibits no ecchymotic areas D. The client exhibits increased serum
albumin level

ANSWER: A

45. A multidisciplinary team has been working with the spouse of Christopher and has been teaching
the spouse interventions for pain management. Which statement by the spouse indicates the need for
further teaching?
A. If the pain increases, I must let the nurse know immediately.
B. I should have my husband try the breathing exercises to control pain.
C. This narcotic will cause very deep sleep, which is what my husband needs.
D. If constipation is a problem, increased fluids will help.

ANSWER: C

SITUATION: Paula, a 21-year-old college student, was admitted at 12 noon because of a generalized
abdominal pain which became localized after midnight on the right lower quadrant accompanied by
nausea and vomiting. In the Emergency Department, the diagnosis of acute appendicitis was confirmed.
Paula was scheduled for Appendectomy.

46. The development of appendicitis usually follows a pattern that correlates with the clinical signs.
The admitting nurse understands that the appendix initially becomes distended with fluid secreted by its
mucosa following:
A. Fibrotic changes in the inner wall of the appendix C. Obstruction of the appendiceal
lumen
B. Impairment of blood supply to the appendix D. Proliferation of microorganism
inside the appendix

ANSWER; C

RATIONALE:

Appendicitis is acute inflammation of the vermiform appendixthe blind pouch attached to the cecum
of the colon that is usually located in the right iliac region, just below the ileocecal valve. Inflammation
of the appendix can occur when the lumen (opening) of the appendix is obstructed. Inflammation
leads to infection as bacteria invade the wall of appendix. When the lumen is
BLOCKED/OSTRUCTED, the MUCOSA CONTINUES TO SECRETE FLUID UNTIL PRESSURE
WITHIN THE LUMEN EXCEEDS VENOUS PRESSUR. Blood flow to the appendix is restricted, and
infection causes more swelling, which further impedes blood flow. Gangrene from hypoxia or
perforation may occurs within 24 to 36 hours and can lead to peritonitis.

OPTION A is INCORRECT.

OPTION B is INCORRECT. Impairment of blood supply will occur as the lumen exceeds venous
pressure. This is the last process in the pathophysiological changes that occurs in appendicitis.

OPTION D is INCORRECT. Bacterial proliferation occurs in the early stage of the disease.

REFERENCE: Ignatavicius and Workmans Medical-Surgical Nursing: Critical Thinking for


Collaborative Care 5th Edition, (2006), Volume 2, p. 1338.

47. The physician noted upon palpation of the Mc Burneys point localized and rebound tenderness.
Which of the following demonstrates this observation?
A. Pain aggravated by coughing
B. Pain increased with internal rotation of the right hip
C. Rigid board-like abdomen
D. Relief of pain with direct palpation and pain on release of pressure

ANSWER: D

RATIONALE:

Abdominal tenderness on palpation is the most common important, and reliable symptoms
related to appendicitis. In later stage if inflammation, tenderness becomes more localized and is
notedwith palpation of the right lower quadrant. This are is referred to as McBurneys point; it is
located midway between the anterior iliac crest and the umbilicus in the right lower quadrant.

OPTIONS A, and C are INCORRECT. These manifestations are indicative of perforated appendix.

OPTION B is INCORRECT. If an inflamed appendix is in contact with the OBTURATOR INTERNUS,


spasm of the muscle can be demonstrated by flexing and internal rotation of the hip. This maneuver
will cause pain in the hypogastrium.

REFERENCE: Ignatavicius and Workmans Medical-Surgical Nursing: Critical Thinking for


Collaborative Care 5th Edition, (2006), Volume 2, p. 1339.
48. Preoperative nursing care plan includes Potential complications related to ruptured appendix as
one of the nursing diagnoses. Which of the following is the nurse expected to report immediately as a
possible sign of ruptured appendix?
A. Severe nausea and vomiting C. Unbearable excruciating localized
pain
B. Sudden increase in body temperature D. Pain subsides

ANSWER: D

RATIONALE:

Diminished pain is the initial manifestation of perforated or ruptured appendix. This is the reason, that
if the definite diagnosis of appendicitis has not been made, administration of analgesic is usually
withheld to prevent in masking the pain.

OPTIONS A, B and C are INCORRECT. These manifestations are related to peritonitis. Presence of
peritonitis also indicates that perforation of inflamed appendix has occurred, however this is usually
seen in the later stage.

REFERENCE: Ignatavicius and Workmans Medical-Surgical Nursing: Critical Thinking for


Collaborative Care 5th Edition, (2006), Volume 2, p. 1339.

49. To prevent perforation of the inflamed appendix, which of the following will the nurse consider as
an effective intervention?
A. Keep on NPO C. Maintain on complete bed rest
B. Monitor progress of pain D. Apply hot compress to abdomen

ANSWER: A

RATIONALE:

OPTION B is INCORRECT. Monitoring the progress of pain will just give you hint if the condition is on
the early stage of later stage. If the pain is still present the condition is considered in the early stage;
however, if the pain subsides it may indicate perforated appendix. This intervention would not prevent
perforation of the inflamed appendix.

OPTION C is INCORRECT. Complete bed rest is not necessarily done to a client with medical
diagnosis of appendicitis

OPTION D is INCORRECT. Hot compress to the abdomen increase the blood supply in the area
because of vasodilation, if the blood supply increases, it can lead to perforation.

REFERENCE: Ignatavicius and Workmans Medical-Surgical Nursing: Critical Thinking for


Collaborative Care 5th Edition, (2006), Volume 2, p. 1340.

50. Postoperative medical diagnosis of the client is Perforated appendix. Client has a nasogastric
tube connected to continuous drainage. Which of the following is the purpose of this intervention?
A. Medium to cleanse the upper GI tract C. Drain out blood
B. Relieve pain due to abdominal distention D. Intestinal decompression

ANSWER: D
RATIONALE:

Perforated appendix is ruptured appendix, meaning if this condition happens outflow of appendiceal
contents into peritoneal cavity occurs and it can cause peritonitis. Usually, if peritonitis is present the
physician may order insertion of NG tube for the main reason of decompressing the intestine.

OPTION A is INCORRECT. Upper GI tract I not affected in clients with appendicitis

OPTION B is INCORRECT. To relieve pain opiod analgesic may be administered

OPTION C is INCORRECT. A drain is usually placed to drain out blood.

REFERENCE: Ignatavicius and Workmans Medical-Surgical Nursing: Critical Thinking for


Collaborative Care 5th Edition, (2006), Volume 2, p. 1340.

SITUATION: Lester has been diagnosed with End-Stage Renal Disease. The physician prescribed
dietary teaching and outpatient hemodialysis three times a week.

51. Lester asks tahe nurse to tell him the purpose of the treatment. Which of the following is the most
appropriate response of the nurse?
A. Hemodialysis removes excess fluids and waste products and restores electrolyte balance.
B. Hemodialysis uses the principles of diffusion and ultrafiltration to remove electrolytes.
C. Blood is pumped through a semipermeable capillary in a hemodialyzer.
D. Hemodialysis is one of several renal replacement therapy.

ANSWER: A

RATIONALE:

Hemodialysis is used for clients who are acutely ill and require short-term dialysis and for cleitns with
advanced CKD and ESRD who requires long-term or permanent renal replacement therapy.
Hemodialysis prevents death but does not cure renal disease and does not compensate for the loss
of endocrine or metabolic activities of the kidneys. The OBJECTIVES/PURPOSES of hemodialysis
are to extract toxic nitrogenous substances from the blood and to remove excess water.

OPTION B is INCORRECT. This statement pertains to the principles on which hemodialysis is based,
but does not specifically discuss the objective of hemodialysis.

OPTION C is INCORRECT. This statement is accurate pertaining to the dialyzer.

OPTION D is INCORRECT. This statement is accurate as hemodialysis along with peritoneal dialysis
and renal transplant are the several renal replacement therapy indicated for clients with ESRD.

REFERENCE: Smeltzer, Suzanne C. Brunner and Suddarths Medical-Surgical Nursing 12th Edition,
(2010), Volume 2, p. 1333.

52. An arteriovenous fistula has been created. Postoperatively, which of the following will the nurse
include as a priority nursing intervention to promote circulation?
A. Auscultate for bruit every 4 hours C. Observe finger tips for cyanosis
B. Elevate the affected arm D. Keep dressing intact

ANSWER: A

RATIONALE:
Auscultate for bruit every 4 hours helps to detect if the AVF is patent in that case this can give the
nurse information regarding the patency of the said fistula. If the graft is not patent, the nurse may
carry out intervention to promote good circulation

OPTION B is INCORRECT. This intervention should be avoided as this may lead to compromise
circulation in the fistula. In addition, the nurse should also advise the client to avoid carrying heavy
objects or anything that compresses the extremity in which the vascular access is placed

OPTION C is INCORRECT. This intervention is necessary to detect presence of complication like


steal syndrome.

OPTION D is INCORRECT. Keeping the dressing intact helps prevents bleeding postoperatively, but
will not promote good circulation.

REFERENCE: Ignatavicius and Workmans Medical-Surgical Nursing: Critical Thinking for


Collaborative Care 5th Edition, (2006), Volume 2, p. 1754.

53. Which of the following pre-dialysis care is done by the nurse to be able to determine the
effectiveness of treatment with regards to excess fluid volume?
A. Assess integumentary status C. Assess vascular site
B. Have patient empty bladder prior to treatment D. Record weight and vital signs

ANSWER: D
RATIONALE:
Assessment of weight and vital signs are essential to evaluate the effectiveness of treatment with
regards to excess fluid volume. Decrease weight and stabilization of vital sign usually indicates that
the treatment was effective.
OPTION A, B and C are INCORRECT. These assessment findings do not specifically assess the
effectiveness of the treatement (hemodialysis) in terms of the fluid status of the client.

54. Nutrition therapy of Lester includes control of protein. Dietary prescription states that Lester is
allowed 0.8 gram of protein per kilogram per day. If Lester weighs 120 lbs, how much is his daily
protein allowance?
A. 57.9 g B. 43.6 g C. 81.7 g D. 96
g

ANSWER: B

RATIONALE:

To compute for daily protein allowance. Take note of the following information that is relevant for the
computation which are the allowable dietary prescription of protein and the weight of the client. The
following steps should be done:

1. First convert the client weight from pounds (lbs) to kilograms (Kgs).
120 lbs/ 2.2 kgs = 54.5 kg
2. Multiply the clients weight in Kilograms to the dietary allowable prescription of protein which
is 0.8 gram.
54.5 kgs x 0.8 = 43.6 g
OPTIONS A, C and D are INCORRECT. These are irrelevant values not related to the given in the
situation.
55. Lester claims he loves to eat raisins. The nurse instructs the patient to avoid this food because it
is rich in which of the following?
A. Sodium B. Potassium C. Magnesium D.
Phosphorus

ANSWER: B

RATIONALE:

Foods rich in potassium are usually avoided in clients receiving hemodialysis. Raisins along with
orange, banana, and avocado are food high with potassium.

OPTIONS A, C and D are INCORRECT.

SITUATION: The kidneys have very important excretory, metabolic, erythropoietic functions. Any
disruptions in the kidneys functions can cause disease. As a nurse, it is important that you understand
the rationale behind the treatment regimen used.

56. P.L. who is in ARF is admitted to the Nephrology unit. The period of oliguria usually lasts for
about 10 days. Which assessment parameter for kidney function will you use during the oliguric phase?
A. Urine output directly related to the amount of IVF infused
B. Urine output is less than 400 mL/24 hours
C. Urine output of 30 60 mL/hour
D. No urine output, kidneys in a state of suspension

ANSWER: B

RATIONALE:
57. During the shock phase, what is the effect of the renin-aldosterone-angiotensin system on renal
function?
A. Increased UO, increased absorption of Na & water C. Increased UO, decreased
absorption of Na & water
B. Decreased UO, decreased absorption of Na & water D. Decreased UO, increased
absorption of Na & water

ANSWER: D

RATIONALE:

58. As you are caring for P.L. who has ARF, one of the collaborative interventions you are expected
to do is to start hypertonic glucose with insulin infusion and sodium bicarbonate to treat:
A. Hyperkalemia B. Hypercalcemia C. Hypokalemia D.
Hypernatremia

ANSWER: A

RATIONALE:

Hyperkalemia is the most life-threatening of the fluid and electrolyte changes that occur in
patients with renal disturbances. Therefore, the patient is monitored for hyperkalemia through
serial serum electrolyte levels (potassium value greater than 5.0 mEq/L [5 mmol/L]), ECG changes
(tall, tented, or peaked T waves), and changes in clinical status. Other symptoms of hyperkalemia
include irritability, abdominal cramping, diarrhea, paresthesia, and generalized muscle weakness.
Muscle weakness may present as slurred speech, difficulty breathing, paresthesia, and paralysis. As
the potassium level increases, both cardiac and other muscular function declines, making this a true
medical emergency. The elevated potassium levels may be reduced by administering cation-
exchange resins (sodium polystyrene sulfonate [Kayexalate]) orally or by retention enema.
Kayexalate works by exchanging sodium ions for potassium ions in the intestinal tract. Sorbitol may
be administered in combination with Kayexalate to induce a diarrhea-type effect (it induces water loss
in the GI tract). If a Kayexalate retention enema is administered (the colon is the major site of
potassium exchange), a rectal catheter with a balloon may be used to facilitate retention if necessary.
The patient should retain the Kayexalate for 30 to 45 minutes to promote potassium removal.
Afterward, a cleansing enema may be prescribed to remove remaining medication as a precaution
against fecal impaction. If the patient is hemodynamically unstable (low blood pressure,
changes in mental status, dysrhythmia), IV dextrose 50%, insulin, and calcium replacement
may be administered to shift potassium back into the cells. Albuterol sulfate (Ventolin HFA) by
nebulizer can lower plasma potassium concentration by 0.5 to 1.5 mEq/L. The shift of
potassium into the intracellular space is temporary, so arrangements for dialysis need to be
made on an emergent basis. In patients with severe acidosis, the arterial blood gases and serum
bicarbonate levels (CO2-combining power) must be monitored because the patient may require
sodium bicarbonate therapy or dialysis.

59. B.N. 40 y/o is diagnosed with CRF. AVF was created for hemodialysis in his left arm. What diet
instructions will you need to reinforce prior to his discharge:
A. Drink plenty of water C. Monitor your fruit intake and eat plenty of
bananas
B. Restrict your salt intake D. Be sure to eat meat every meal

ANSWER: A

60. B.N. is also advised not to use salt substitute in the diet because:
A. Salt substitute contain potassium which must be limited to prevent arrhythmias
B. Limiting salt substitute in the diet prevents a buildup of waste products in the blood
C. Fluid retention is enhanced when salt substitutes are included in the diet
D. A substance in the salt substitute interferes with fluid transfer across the capillary membrane

ANSWER: A

RATIONALE:

Salt substitute contains potassium which is contraindicated in a client with renal failure

OPTION B is INCORRECT. Limiting the salt substitute would not help prevent buildup of waste
products.
OPTION B is INCORRECT. Again, salt substitute enhances the potassium level in the body which is
contraindicated in renal failure

SITUATION: You are assigned in the nephrology ward. One of your patients is Jerry with an admitting
impression of right renal calculi. Based on his history, Carlo was brought to the ED by his friend when he
experienced severe excruciating right flank pain, nausea and vomiting. This was relieved when the doctor
gave him Buscopan. The doctor ordered several diagnostic work-up. Kidney urinary bladder and
Intravenous Pyelography (KUB-IVP) and ultrasound. Blood chemistry and 24 hour urine collection to
measure calcium, uric acid, creatinine, sodium, pH and total volume were likewise ordered.

61. As the nurse of Carlo who is for KUB-IVP, which of the following will you include in your teaching
plan?
1. What is an intravenous pyelography (IVP)?
2. How should I prepare for the procedure?
3. How is the procedure performed?
4. What will I experience during and after the procedure?
A. 3 and 4 B. 1 and 2 C. All except 3 D. All
of the above

ANSWER: D

RATIONALE:

The client in the situation is admitted with an admitting impression of renal calculi (stone). To help in
establishing the diagnosis, the physician may order various laboratory tests. One of those tests is the
KUB-IVP (Kidney, Ureters and Bladder-Intravenous pyelography) In KUB it involves x-ray of the said
organs, while in IVP, contrast medium is injected intravenously, following the injection or instillation of
the contrast medium, x-rays are taken to evaluate urinary tract structure. In clients that will undergo a
certain procedures that are not familiar to them, it is essential to give pertinent information from the
pre-procedure up to the post procedure. This will somehow alleviate the anxiety that the client may
feel. Therefore, statements on No. 1, 2, 3, and 4 should be included when making the teaching plan
regarding KUB-IVP.

OPTIONS A, B and C are INCORRECT. These are also accurate however, they are incomplete as
compare to option D.

REFERENCE: Koziers Fundamentals of Nursing 8th Edition, (2008), Volume 2, p. 817.

62. One of the physicians orders is a 24 hour urine test. What instructions will you give Carlo if the
collection will start tomorrow at 7:00 AM (Day 1) and end at 7:00 AM the following day (Day 2)?
A. Discard your first urine sample at 7:00 AM tomorrow then start urine collection until 7:00 AM Day
2
B. Start urine collection at exactly 7:00 AM tomorrow up to Day 2 but discard the last urine at 7:00
AM of Day 2
C. Discard your first urine sample tomorrow at 7:00 AM and your last urine sample collection on Day
2 at 7:00 AM and collect all urine samples between those times
D. Start urine sample collection at exactly 7:00 AM tomorrow

ANSWER: A

RATIONALE:
Some urine examination requires collection of all urine produced and voided over a specific period of
time ranging from 1 to 2 hours to 24 hours. Timed specimens generally either are refrigerated or
contain a preservative to prevent bacterial growth or decomposition of urine component. Some of the
test performed on timed urine specimens includes the following purposes:

To assess the ability of the kidney to concentrate and dilute urine


To determine disorders of glucose metabolism, for example diabetes mellitus
To determine levels of specific constituents, for example, albumin, amylase, creatiine,
urobilinogen, and certain hormone.
To collect a timed urine specimen, follow these steps:

Obtain a specimen container with preservative (if indicated) from the laboratory. Label the
container with identifying information for the client, the test to be performed, time starter and
time of completion
Provide a clean receptacle to collect urine (bedpan, commode or toilet collection device)
Post sign in the clients chart, Kardex, room and bathroom alerting personnel to save all urine
during the specified time.
At the START of collection period, have the client void and DISCARD this urine.
Save all urine produced during the timed collection period in the container, refrigerating or
placing the container on ice as indicated. Avoid contamination the urine with toilet paper or
feces
At the end of the collection period, instruct the client to completely empty the bladder and
SAVE this voiding as part of specimen. Take the entire amount of urine collected to the
laboratory with completed requisition.
OPTIONS B, C and D are INCORRECT. These statements regarding the collection of timed urine
specimen are inaccurate and irrelevant.

REFERENCE: Koziers Fundamentals of Nursing 8th Edition, (2008), Volume 2, p. 817.

63. The diagnostic examination confirmed the presence of renal calculi. A dietary medication and
family history of renal stone was part of the assessment that was done in order to:
A. Avoid taking drugs that could have contributed to stone formation
B. Identify the factors predisposing Carlo to formation of stone
C. Prescribe the type of diet that is needed to prevent recurrence
D. Identify what type of stone was formed

ANSWER: B

RATIONALE:

The nurse obtains some data regarding the clients dietary regimen and family history primarily to
assess factors that could have contributed to the development of his condition, which is renal calculi.

OPTION A is INCORRECT. In this statement, the nurse should have had assessed the client
medication history.

OPTIONS C and D are INCORRECT. These statement is related only to dietary regiment of the
client, but not the family history.

64. To facilitate spontaneous passage of stone and dilute the urine, the following can be advised for
Carlo, except:
A. Limit fluid intake to 1.5 liters per day to avoid fluid overload
B. Examine all urine output for presence of stone
C. Promote sufficient fluid intake to maintain a urine output of 3-4 liters per day
D. Record intake and output and daily weight to assess fluid status and renal function

ANSWER: A

RATIONALE:

Limiting the clients fluid intake would concentrate his urine. The more concentrated the urine is the
more stone is being formed. Therefore, the nurse should instruct the client to avoid this intervention.
Instead, the nurse should encourage the client to drink at least 3-4 liters daily.

OPTION is INCORRECT. This intervention will not directly facilitate spontaneous passage of stone.
However, the mere fact that the nurse assess the urine output for the presence of stone would help
determine if stones are passing or being excreted out of the clients urinary system. Moreover,
assessment would lead you to intervention.

OPTION is INCORRECT. As stated earlier, adequate fluid intake is essential in facilitating passage of
stone as well as preventing further stone formation.

OPTION is INCORRECT. Recording intake and output will also give findings if the client is passing
stone out of the urinary tract. Therefore, as stated on option B this intervention is also essential.

REFERENCE: Smeltzer, Suzanne C. Brunner and Suddarths Medical-Surgical Nursing 12th Edition,
(2010), Volume 1, p. 1377.

65. Stone assay was done following its spontaneous passage. The result revealed uric acid stone.
Prior to his discharge, you made a teaching plan for Carlo on how he can prevent kidney stone
formation. The following were included in your plan, except:
A. Take Allopurinol (Zyloprim) that is prescribed by the doctor to reduce serum uric acid levels and
urinary uric acid secretion
B. Avoid foods high in purine like shellfish, anchovies, mushroom and organ meat
C. During the waking hours drink fluids every 1-2 hours and at bedtime take 2 glasses of water to
prevent urine from being too concentrated
D. Encourage activities leading to sudden increase in temperature to facilitate excretion of uric acid
through sweating

ANSWER: D

RATIONALE:

Sudden increase in temperature causes excessive sweating and subsequently dehydration.


Remember that renal stones are easily formed when a client is dehydrated. Therefore, the nurse
should advise the client regarding this matter.

OPTION is INCORRECT. The clients urinary stone as stated in the question is mainly composed of
uric acid. Therefore, medication that prevents production of uric acid should be given to prevent
further stone formation. This medication is Allopurinol (Zyloprim).

OPTION is INCORRECT. Foods high in purine does contribute in uric acid stone formation as this
food items have high uric acid content

OPTION is INCORRECT. Drinking two glasses of water at bedtime and an additional glass at each
nighttime awakening helps prevent urine from becoming too concentrated during night. In addition,
during waking hours, the client may be instructed to drink every 1-2 hours.
REFERENCE: Smeltzer, Suzanne C. Brunner and Suddarths Medical-Surgical Nursing 12th Edition,
(2010), Volume 1, p. 1377.

SITUATION: Antonio, age 52, was admitted to the hospital with acute adrenal insufficiency. He has a
history of Addisons disease for which he has been taking hydrocortisone. Over the past week, he has
flulike symptoms accompanied by nausea and vomiting. When he awoke this morning, his wife noticed
that he was confused and extremely weak, so she brought him to the hospital for evaluation.

66. The nurse would assess for which signs and symptoms for a client diagnosed with addisonian
crisis?
A. Polyuria, polydipsia, and polyphagia C. Hypotension, rapid respirations, and
pallor
B. Tremors, tachycardia, and headache D. Positive Chvostek's sign, photophobia,
and numbness

ANSWER: C

RATIONALE:

Addisons disease, or adrenocortical insufficiency, occurs when adrenal cortex function is inadequate
to meet the patients need for cortical hormones. Autoimmune or idiopathic atrophy of the adrenal
glands is responsible for the vast majority of cases. Other causes include surgical removal of both
adrenal glands and infection of the adrenal glands. Tuberculosis and histoplasmosis are the most
common infections that destroy adrenal gland tissue. With disease progression and acute
hypotension, addisonian crisis develops. This condition is characterized by:

Cyanosis and the classic signs of circulatory shock: pallor, apprehension, rapid and weak
pulse, rapid respirations, and low blood pressure.
In addition, the patient may complain of headache, nausea, abdominal pain, and diarrhea and
may show signs of confusion and restlessness.
Even slight overexertion, exposure to cold, acute infection, or a decrease in salt intake may lead to
circulatory collapse, shock, and death if untreated. The stress of surgery or dehydration resulting from
preparation for diagnostic tests or surgery may precipitate an addisonian or hypotensive crisis.

OPTION A is INCORRECT. These manifestations are related to diabetes mellitus

OPTION B is INCORRECT. These manifestations are related to hyperthyroidism

OPTION D is INCORRECT. These manifestations are related to hypoparathyroidism

REFERENCE: Brunner and Suddarths Medical and Surgical Nursing 12th Edition, (2010), Volume 2,
p. 1279

67. Antonios blood pressure is 90/58 mmHg, his heart rate is 116 bpm, and his temperature if 101 oF.
The nurse should expect to start an IV infusion of:
A. Insulin B. Hydrocortisone C. Potassium D.
Hypotonic saline

ANSWER: B

RATIONALE:

Immediate treatment is directed toward combating circulatory shock: restoring blood circulation,
administering fluids and corticosteroids, monitoring vital signs, and placing the patient in a recumbent
position with the legs elevated. Hydrocortisone (Solu-Cortef) is administered by IV, followed by
5% dextrose in normal saline. Vasopressor amines may be required if hypotension persists.

OPTION A is INCORRECT. In addisonian crisis, sugar level is decreased, if we give insulin it can
further causes depletion of sugar level which can cause dangerous hypotension

OPTION C is INCORRECT. Potassium level of a client with addisonian crisis is elevated. In that case,
the nurse should limit the intake of potassium.

REFERENCE: Brunner and Suddarths Medical and Surgical Nursing 12th Edition, (2010), Volume 2,
p. 1279

68. Which is an appropriate measure for a patient with addisonian crisis?


A. IV fluid replacement B. IV corticosteroids C. Blood glucose management D. All
of the above

ANSWER: D

RATIONALE:

As stated earlier, treatment is directed in combating the circulatory shock. Interventions that are
indicated are the following: IV fluid replacement and IV corticosteroid. In addition, in clients with
addisonian crisis, blood sugar level is affected, commonly they suffer from hypoglycemia, in that case
blood glucose management is also indicated.

REFERENCE: Brunner and Suddarths Medical and Surgical Nursing 12th Edition, (2010), Volume 2,
p. 1279

69. During the initial 24 hours after admission, the nurse should frequently:
A. Weigh the patient C. Assess vital signs
B. Test the patients urine for ketones D. Administer oral hydrocortisone

ANSWER: C

RATIONALE:

Clients with addisonian crisis are at risk to develop circulatory shock, meaning they are unstable. The
nurse should assess for vital signs as well as fluid status every 30 minutes for the first 24 hours until
stable.

OPTION A is INCORRECT. Weight is also important however, assessment of the vital signs and fluid
status takes priority

OPTION B is INCORRECT. Ketones shouldnt be present, because clients with addisonian crisi often
develop hypoglycemia

OPTION D is INCORRECT. Since this is an acute condition, clients should receive parentera; form
(IM form) of hydrocortisone.

70. The patient in addisonian crisis is unable to respond to stress. Before discharge, the nurse should
instruct Antonio and his family that during stress itll be necessary to:
A. Administer cortisone IM C. Perform capillary blood glucose
monitoring four times daily
B. Drink 8 oz of fluids D. Continue to take his usual dose of
hydrocortisone

ANSWER: A

RATIONALE:

Stress is a precipitating factor in the development of crisis. To prevent crisis from development, the
nurse should instruct the client to administer cortisone IM.

OPTION B is INCORRECT. This intervention is necessary when crisis ensue as this intervention
prevents shock. However, the question is asking for intervention that will prevent crisis from
happening during stressful situation.

OPTION C is INCORRECT. This intervention is also necessary, however would not help in preventing
crisis.

OPTION D is INCORRECT. Dosage modification is necessary if there is stressful situation.

NP5 (1-40)

SITUATION: Nurse Rose has three discharged client during the early part of the P.M. shift. The senior
nurse assigned her to take charge of any admission during the shift. A lethargic female client came in for
thyroid work-up.

1. As prescribed by the attending physician, the nurse instructed the client to undergo Radioactive
Iodine Uptake Test the following morning. The client asks to be educated on the test. Nurse Rose
would explain that the purpose of the test is to:
A. Demonstrate the extent of damage/compression rendered by the nodule to the trachea
B. Detect if the thyroid nodule is malignant or benign
C. Determine the functional activity of the thyroid gland and differentiate pituitary from thyroid
function
D. Measure the ability of the thyroid gland to remove and concentrate iodine from the blood

ANSWER: D

RATIONALE:

The radioactive iodine uptake test (RAIU) measures the ability of the thyroid gland to remove and
concentrate iodine from the blood. This test is done to find problems with how the thyroid gland
works, such as in hyperthyroidism and hypothyroidism. Clients with hyperthyroidism may exhibit a
high uptake of iodine, while those clients with hypothyroidism may exhibit a very low uptake.

To prepare for this test, the following should be taken into consideration:
Take any medicines regularly. Be sure your doctor knows the names and doses of all your
medicine. Your doctor will instruct you if and when you need to stop taking any medication of
the following medicines that can change the RAIU test result:
Thyroid hormone medication
Antithyroid medication
Medicine that contain iodine, such as iodize salt, kelp, cough syrups, multivitamin or
heart medication amiodarone
Are allergic to any medicines, such as iodine. But even if you are allergic to iodine, you will
likely be able to have this test because te amount used in the tracer is so small that your
chance of an allergic reaction is very low.
Have had any test using radioactive materials or iodine dye 4 weeks before the RAIU test.
These other test may change the result of the RAIU test.
Are or might be pregnant (usually contraindicated with RAIU test)
Are breast- feeding (usually contraindicated with RAIU test)
Do not eat for 2 hours before the test
Stop taking antithyroid medication for 5 to 7 days before the test
Eat low-iodine diet
OPTION C is INCORRECT. This statement is related to thyroid function test.

OPTION B is INCORRECT. This statement is related to fine-needle aspiration biopsy to the thyroid
gland

OPTION A is INCORRECT. This statement is related to tracheal stenosis imaging

REFERENCE: Smeltzer, Suzanne C. Brunner and Suddarths Medical-Surgical Nursing 12th Edition,
(2010), Volume 2, pp. 1255-156.

2. Before the radioactive iodine uptake test, the nurse should verify which of the following would affect
the result of the test?
A. Over-the-counter drug intake C. Height and weight
B. Sleeping habits D. Food preference

ANSWER: A

RATIONALE:

A lot of over the counter medications contains iodine. These medications include cough syrups,
bronchodilators, estrogens, salicylates, lithium, amniodarine, propanolol

OPTIONS B, C and D are INCORRECT. Sleeping habits, height and weight, food preference would
not alter the result

REFERENCE: Smeltzer, Suzanne C. Brunner and Suddarths Medical-Surgical Nursing 12th Edition,
(2010), Volume 2, p. 156.

3. The doctor prescribed levothyroxine sodium 0.15 mg per orem daily after the diagnosis of
hyperthyroidism was confirmed. Nurse Rose administers the medication at which time to obtain the
drugs optimum therapeutic level?
A. In the morning before breakfast C. At various times of the day
B. At the patients most convenient time D. Before bedtime

ANSWER: A
RATIONALE:

As you look back to the situation, the client is lethargic. In that case we night say that the client may
be experiencing hypothyroidism. Client with this condition requires life-long therapy of thyroid
hormone, the most commonly prescribed is Levothyroxine (Synthroid). When giving this medication,
the following should be taken into consideration:

Thyroid hormones enhances the action of oral anticoagulants, sympathomimetics, and


antidepressant and decrease the action of insulin, oral hypoglycemic and digitalis
preparation; the action of thyroid hormones is decrease by phenytoin andcarbamazepine
Thyroid hormones should be given at least 4 hours apart from multivitamins, aluminum
hydroxide and magnesium hydroxide, simethicone, calcium carbonate, iron, and sucralfate
because these medication decrease the absorption of thyroid replacements.
To obtain the drug optimum therapeutic level, the nurse should administer this medication in the
morning before breakfast. Besides that, administering this medication during day prevent the
occurrence of insomnia which is a common side effect.
Avoid foods that can inhibit thyroid secretion such as strawberries, peaches, pears,
cabbage, turnips spinach, cauliflower, radishes and peas.
Monitor for side effects:
Nausea and decrease appetite
Abdominal cramps and diarrhea
Weight loss
Nervousness and tremors
Insomnia
Sweating and heat intolerance
Tachycardia, dysrhythmias, palpitation and chest pain
Hypertension
Headache
TOXICITY: MANIFESTATION OF HYPERTHYROIDISM
OPTIONS B and D are INCORRECT. These statements are inaccurate. Remember, every
medication should be taken considering the time.

OPTION D is INCORRECT. If this medication is taken during bedtime, the client may experience
insomnia, therefore, it should be avoided to be taken during bed time.

REFERENCE: Silvestri, Linda Anne. Saunders Comprehesive Review for the NCLEX-RN
EXAMINATION 4TH Edition, Chapter 54, pp. 3-6.

4. Nurse Rose included in her health instruction the foods that inhibit thyroid secretions and this includes
which of the following?
1. Spinach 3. Squash 5. Strawberries
2. Cauliflower 4. Radish 6. Guavas
A. All except 3 and 6 C. All except 2 and 4
B. All of these D. All except 1 and 5

ANSWER: A

RATIONALE:

Food items that inhibit thyroid secretions include spinach (1), cauliflower (2), radish (4), strawberries
(5). Others are peaches, pears, cabbages, turnips, and peas. Therefore, options that contain
statements on 1, 2, 4 and 5 are correct (All except 3 and 6 is the same as 1, 2, 4 and 5).

OPTIONS B, C and D are INCORRECT. Please refer to the explanation above.


REFERENCE: Silvestri, Linda Anne. Saunders Comprehensive Review for the NCLEX-RN
EXAMINATION 4TH Edition, Chapter 54, p. 6.

5. Nurse Rose would include in her discharge plan for the client and significant others the regular intake
of which product that would ensure iodine intake?
A. Lugols solution C. Warm salt solution gargle
B. Seafood D. Iodized salt

ANSWER: B

RATIONALE:

As a component if the thyroid hormone thyroxine (T4) and triiodothyroixine (T3), iodine is essential to
human life. Without sufficient iodine, you body is unable to synthesize these hormones, and because
the thyroid hormones regulate metabolism in every cell of the body and play a role in virtually all
physiological function, an iodine deficiency can have a devastating impact on the health and well-
being. In the past, iodize salt was encourage to take to ensure adequate iodine intake, however the
WHO (2007) is exploring alternative strategies to ensure ioditne intake because of the health risks
associated with excessive salt intake. Seafoods especially sea vegetables contains 4,5000ug of
iodine.

OPTION A is INCORRECT. Lugols solution is used in clients with hyperthyroidism. This medication
is usually administer prior to thyroidectomy as this medication decrease vascularity of the thyroid
hormone at the same time this medication also suppress the secretion of thyroid hormone to prevent
thyroid storm during the surgical procedure.

OPTION C is INCORRECT. Warm salt solution gargle would not provide adequate iodine intake.

OPTION D is INCORRECT. Fortified iodize sat only contains 77 ug of iodine.

SITUATION: Antonina, a newly hired staff nurse in the medical-surgical unit, was assigned to work with a
senior nurse. A female client was admitted with a diagnosis of diabetic foot, gangrene left toe, type 2
diabetes. Antonina assisted the senior nurse during the admission of the client.

6. From the nursing history obtained from the client, which information is most likely related to the
development of gangrene on the clients left toe?
A. Preferred open toed sandals to closed leather shoes
B. Father had type 2 diabetes, post above knee amputation, right leg
C. Type 2 diabetes mellitus diagnosis 15 years ago
D. Accidental cut on big toe while cutting toenails

ANSWER: C

RATIONALE:
7. The physician ordered bilateral lower extremities Doppler ultrasound. Which of the following is the
physician interested to find out through this diagnostic test?
A. Distal paresthesias C. Occlusion of large vessels and
arterioles
B. Oxygenation of tissues in the lower extremities D. Isolated peripheral neuropathies

ANSWER: C RATIONALE:
8. The senior nurse asked Antonina to list nursing interventions for the nursing diagnosis Ineffective
tissue perfusion: peripheral. From the following list prepared by Antonina, which intervention will the
senior nurse consider to be contraindicated?
A. Regular passive and active exercises of all extremities C. Encourage frequent change in
positions
B. Keep extremities warm using a foot cradle D. Maintain both extremities in a
dependent position

ANSWER: B

RATIONALE:

Foot cradle are used to elevate the lower extremities. Elevation of lower extremities further decrease
the blood flow thus aggravating the ineffective tissue perfusion in the lower extremities (peripheral)

OPTIONS A, C and D are INCORRECT. All of these are appropriate interventions to manage a client
with ineffective tissue perfusion

9. When Antonina checked the capillary blood glucose of the client at 6 pm before meals as instructed
by the senior nurse, the result showed 65 mg/Dl. Which of the following will Antonina do first?

A. Give juice as prescribed in the Insulin scale pre-meals


B. Check the physicians order in case CBG is below 70 mg/dL
C. Recheck the CBG
D. Look for the senior nurse and report

ANSWER: C

RATIONALE:

10. The senior nurse observes that Antonina occasionally does not follow agreed upon interventions. The
senior nurse reports that Antonina improves in which of the following?
A. Identifying own learning needs C. Compliance to standards
B. Attitude towards criticism D. Demonstration of proper decorum

C. ANSWER: C

SITUATION: Jessie and James, scrub nurse and circulating nurse, respectively, are preparing for
hydrocoelectomy their last case for the day.

11. Jessie discovered a cut in her palm while she was opening the sterile packs. Jessie should:
A. Scrub but put on double gloves C. Scrub if the cut is properly bandage
B. Ask to be relieved as scrub nurse D. Scrub after writing an incident report

ANSWER: A

RATIONALE:
In the situation, Jessie (scrub nurse), is part of the sterile team. Under the principle of sterile
technique, person touching sterile items should also be sterile. In this situation, Jessie had a cut in
her palm while opening the sterile packs; this situation may cause contamination of the sterile item.
What Jessie needs to do is that, she needs to scrub and put on double gloves to prevent
contamination of the sterile items

OPTION B is INCORRECT. This situation is easily manageable. There is no need for change of
scrub nurse.

OPTION C is INCORRECT. Bandage alone would not help ensure that Jessie is sterile, scrubbing
and putting a double glove would help to do so.

OPTION D is INCORRECT. Writing an incident report is not indicated in this situation. This is just a
matter of sterility, besides no client is injured.

12. Before any member of the sterile surgical team proceed to do the surgical hand scrub, she/he should
have complete operating room (OR) attire. Identify all the attire appropriate for this case.
1. Head cap 3. Face mask 5. Gloves
2. Goggles 4. Sterile gown
A. 2, 3, 4 B. 1, 3, 4 C. 1, 2, 3 D. 3,
4, 5

ANSWER: C

RATIONALE:

Before surgical hand scrubbing, the scrub nurse should first wear appropriate OR attire, which
consists of head cap, goggles and face mask.

OPTIONS A, B and D are INCORRECT. Sterile gown and gloves are usually worn inside the
operating room, and after surgical hand scrubbing

13. A sterile set-up has been prepared. The OR was notified of a delay in transporting the client from
the ward to the OR. Which of the following guidelines should the circulating nurse follow?
A. Keep door of the operating room closed all the time to maintain sterile set-up
B. Sterile set-up should be replaced after an hour
C. Cover appropriately the sterile set-up
D. Prepare another sterile set-up

ANSWER: A

14. Identify which appropriate gloving technique will the surgeon use when he performs the skin
preparation?
A. Gloving self-closed technique C. Scrub nurse serves the gloves
B. Gloving self-open technique D. Any gloving technique is accepted

ANSWER: B

RATIONALE:
When doing skin preparation as well as urinary catheterization, the appropriate gloving technique
would be open gloving technique

REFERENCE: Surgical Technology Review: Certification & Professionalism by Karen L Chambers.

15. The intern-2nd assistant surgeon contaminated his gown while the surgery is ongoing. He is expected
to change his gown and gloves. Which of the following is the CORRECT technique to be followed?
A. The intern removes his gown and gloves then puts on another sterile gown and gloves
B. The intern removes his gloves, then his gown, does a 3 minutes hand scrub and don another
sterile gown and gloves
C. The intern unties his gown removes his gowns and put on another gown and gloves
D. The circulating nurse unties the gown. The intern removes his gown, then removes the gloves
and puts another sterile gown and gloves

ANSWER: D

RATIONALE:

Occasionally, the sterile gown becomes contaminated during the procedure. In this case, both the
gown and the gloves should be replaced. If there are indication that the arms/hands also becomes
contaminated then a surgical scrubbing should be repeated. However, in particular to this question
there are no cues regarding the matter. In that case scrubbing is not necessary. The proper
techniques are as follow:

The contaminated member of the sterile team steps away from the sterile field to allow the
circulator access to the gown
The circulator pulls the gown forward off the shoulders, bundling it inward to contain the front,
which has been exposed to blood and body fluids
Another team member may regown the person, or he or she can gown and glove themselves
using closed technique.
REFERENCE: Surgical Technology: Priciples and Practice by Joanna Kotcher Fuller p. 190

SITUATION: Chris, a 30 year old bank employee, suffered a head trauma after he was hit by a blunt
object leaving him unconscious.

16. He regains consciousness while he was in the hospital. While the nurse is doing assessment, the
client verbalizes ringing noises. Anatomically, the complaints suggest injury of the:

A. Occipital lobe B. Vestibular nerve C. Abducens nerve D.


Frontal lobe

ANSWER: B

RATIONALE:

Hearing discomforts and difficulties following a head trauma may be indicative of vestibular nerve
damage.

OPTION A is INCORRECT. Injury in the occipital lobe following a head trauma may cause visual
impairment to the client as this is the visual perception center.

OPTION C is INCORRECT. Injury in the abducens nerve may cause diplopia

OPTION D is INCORRECT. Frontal lobe controls motor skills like hand/eye coordination, conscious
thought, emotions and even the personality. As a result of brain injury, frontal love damage may
impair the attention span, motivation, judgment and organizational capacity
17. Soon after admission to the hospital, the client manifested a combination of symptoms indicative of
increased intracranial pressure. Of the following signs and symptoms which of the following are the
MOST indicative of increased intracranial pressure?
A. Slow bounding pulse rising systolic blood pressure, elevated temperature, and stupor
B. Weak rapid pulse, normal blood pressure, intermittent fever and lethargy
C. Rapid bound pulse, rising blood pressure and elevated temperature
D. Rapid weak pulse, lowered body temperature and increased blood pressure

ANSWER: A

RATIONALE:

Increased intracranial pressure (ICP) occurs when an increase in any one of the components causes
a change in the volume of the others. Because brain tissues has limited space to expand,
compensation typically is accomplished by displacing or shifting of CSF, increasing the absorption or
diminishing the production of CSF or decreasing cerebral blood volume. Without such changes, ICP
begins to rise. Increase ICP may reduce cerebral blood flow, resulting in ischemia and cell death. In
the early stages of cerebral ischemia, the vasomotor centers are stimulated and systemic pressure
rises to maintain cerebral blood flow. Slow bounding pulse and respiratory irregularities as well as
increase BP and temperature may be observed. Decrease or alteration in the clients level of
consciousness may also be present (earliest manifestation).

OPTIONS B, C and D are INCORRECT. These manifestations are not related to increase ICP.

REFERENCE: Smeltzer, Suzanne C. Brunner and Suddarths Medical-Surgical Nursing 12th Edition,
(2010), Volume 2, p. 1865.

18. In caring for the client with possible skull fracture as a result of head trauma, the nurse should:
A. Elevate the foot of the bed to prevent shock C. Observe of changes in respiratory
changes
B. Monitor vital signs and blood pressure D. Observe for signs of brain injury

ANSWER: D

RATIONALE:

Clients with skull fracture may develop brain injury. Therefore, it is necessary for the nurse to observe
a client with skull fracture regarding manifestations of brain injury

OPTION A is INCORRECT. Elevation of foot of the bed to prevent shock is necessary if too much
blood is loss. However, there are no cues that will support this statement

OPTION B is INCORRECT. Monitoring vital signs and blood pressure is also important however,
detection of brain injury is the priority

OPTION C is INCORRECT. Again, this is also important in the care of a client with skull fracture, but
observation of the development of brain injury is the priority at this moment.

19. Chirs had a craniotomy. After the client returns from surgery, the nurse must know that the optimum
positioning of a neurosurgery client, unless otherwise indicated would be;
A. Flat on bed C. Head elevated at 80 90 degrees
B. Head elevated at 20 30 degrees D. Lateral or prone position
ANSWER: B

RATIONALE:

The client in the situation has undergone supratentorial surgery typically after the surgery, the nurse
should place the client to a position that will prevent increase in ICP. The nurse should place the
client with head of bed elevated at least 30 degrees as this position provides optimum drainage of
CSF, preventing or halting increase ICP.

OPTION A is INCORRECT. Placing the client flat on bed following cranial surgery may further
increase ICP

OPTION C is INCORRECT. As stated earlier, the head should be elevated at least 30 degress, not
80 -90 degrees.

OPTION D is INCORRECT. This position is indicated to client who undergone infratentorial surgery.

REFERENCE: Smeltzer, Suzanne C. Brunner and Suddarths Medical-Surgical Nursing 12th Edition,
(2010), Volume 2, p. 1877.

20. Following the immediate post operative period, the nurse should assess the client for:
A. Constricted pupils C. Rapid weak pulse
B. Elevated diastolic blood pressure D. Decreased level of consciousness

ANSWER: D

RATIONALE:

The major complication that may arise during postoperative period is the increase in ICP. In
monitoring a client about manifestations related to increase ICP, the nurse should observe alteration
in the clients level of consciousness as this is the earliest manifestation of increase ICP

OPTION A is INCORRECT. Dilated pupils rather than constricted pupils is seen in clients with
increase ICP

OPTION B is INCORRECT. Elevated systolic pressure is seen in clients with increase ICP

OPTION C is INCORRECT. Slow bounding pulse is seen instead of rapid weak pulse.

REFERENCE: Smeltzer, Suzanne C. Brunner and Suddarths Medical-Surgical Nursing 12th Edition,
(2010), Volume 2, p. 1865.

SITUATION: Upon discharge, the patient with Chronic Obstructive Pulmonary Disease (COPD) requires
considerable patient and family teaching.

21. A nurse instructs a client diagnosed with COPD to use purse-lip breathing. The client inquires the
nurse about the advantage of this kind of breathing. The nurse answers that the main purpose of
purse-lip is to:
A. Prevent bronchial collapse C. Achieve maximum inhalation
B. Strengthen the intercostals muscle D. Allows air trapping
ANSWER: A

RATIONALE:

The breathing pattern of most clients with COPD is shallow, rapid, and inefficient; the more severe
the disease, the more inefficient the breathing pattern. Clients with COPD are advised to do pursed-
lip breathing exercise because this breathing exercise helps:

Slow expiration
PREVENT COLLAPSE OF SMALL AIRWAYS
Control the rate and depth of respiration
Promotes relaxation, enabling the client to gain control of dyspnea
Here are the steps to follow when doing pursed-lip breathing exercise:

Inhale through the nose while slowly counting to 3 the amount of time needed to
say smell a rose.
Exhale slowly and evenly against pursed lips while tightening the abdominal muscles
Count to 7 slowly while prolonging expiration through pursed lips
While sitting in a chair:
Fold arms over the abdomen
Inhale through the nose while counting to 3 slowly
Bend forward and exhale slowly through pursed lips while counting to 7
slowly
While walking:
Inhale while walking two steps
Exhale through pursed lips while walking four to five steps
OPTION B is INCORRECT. Pursed-lip breathing does not strengthen the intercostals muscle.

OPTION C is INCORRECT. As stated above, pursed-lip breathing slow expiration than helps to
achieve maximum expiration of carbon dioxide.

OPTION D is INCORRECT. Air trapping is a problem in COPD clients because of over distended
alveoli (emphysema) and mucus plugging in the airway (chronic bronchitis). Pursed lip breathing is
advised to a client with COPD not primarily to allow air trapping but to prevent bronchial collapse as
stated earlier.

REFERENCE: Smeltzer, Suzanne C. Brunner and Suddarths Medical-Surgical Nursing 12th Edition,
(2010), Volume 1, pp. 602 & 603, 611 & 612, 641.

22. The nurse teaches a patient about the use of respiratory inhaler. Arrange the steps in using an inhaler
chronologically.
1. Press the canister down with your fingers as you breathe in
2. Wait one minute between puffs if more than one puff is prescribed
3. Inhale the mist, hold your breath at least 5 to 10 seconds before exhaling
4. Remove the cap and shake the inhaler
A. 4, 1, 2, 3 B. 3, 4, 2, 1 C. 4, 1, 3, 2 D. 1,
2, 3, 4

ANSWER: C

RATIONALE:

A respiratory inhalers (metered-dose inhalers [MDI]), is a device that delivers a specific amount of
medication to the respiratory system. It is the most commonly used delivery system for treating
asthma, and COPD. To use the nurse should instruct the client about the following steps:
Before each use, Remove the cap and shake the inhaler according to the instruction
in the package insert (4)
Tilt the head back slightly and breathe out fully
Open your mouth and place the mouth piece 1 to 2 inches away
As you begin to breathe in deeply through the mouth, press down firmly on the
canister of the inhaler to release one dose of medication (1)
Continue to breath in slowly and deeply (usually over 3 to 5 seconds)
Hold your breath for at least 5 to 10 seconds to allow the medication to reach deep
into the lungs, then breathe out slowly (3)
Wait at least 1 minutes between puffs (2)
Replace the cap on the inhaler
At least once a day, remove the canister and clean the plastic case and cap of the
inhaler by thoroughly rinsing in warm, running tap water
OPTIONS A, B and D are INCORRECT. Please refer to the explanation above.

REFERENCE: Ignatavicius and Workmans Medical-Surgical Nursing: Critical Thinking for


Collaborative Care 5th Edition, (2006), Volume 1, p. 593.

23. The physician prescribed monitoring closely of clients oxygen saturation of the blood. Which of the
following will you prepare?
A. Electrocardiogram machine C. Pulse oximeter
B. Spirometer D. Blood Pressure apparatus

ANSWER: C

RATIONALE:

Pulse oximeter is a non-invasive device measures arterial blood oxygen saturation by means of
sensor attached to the clients finger, toe, nose, earlobe or forehead (or around the hand or foot of a
neonate). The pulse oximeter can detect hypoxemia before clinical signs and symptoms, such as
dusky color and dusky nail beds develop. Normal SaO2 is 95% to 100% and SaO2 below 70% is life
threatening. NOTE: if the client has nail polish, removed it before applying the sensor because it can
interfere with the result. In addition, the pulse oximeter should be covered from external source of
light because it can also interfere with the result.

OPTION A is INCORRECT. Electrocardiogram (ECG) measure or records the electrical activity of the
heart

OPTION B is INCORRECT. Spirometer/ Incentive spirometers (sustained maximal inspiration devices


[SMIs]), is used to measure the flow of air inhaled through the mouth piece and are used to:

Improve pulmonary ventilation


Counteract the effects of anesthesia or hypoventilation
Loosen respiratory secretions
Facilitate respiratory gaseous exchange
Expand collapsed alveoli,
To use, the nurse may instruct the client to do the following:

Assume a sitting or upright position


Place the mouth tightly around the mouthpiece
Inhale slowly to raise and maintain the flow rate indicator between the 600 and 900 marks
Hold breath for 5 seconds and then to exhale through pursed lips
Repeat this process 10 times every hour
OPTION D is INCORRECT. Blood pressure apparatus measure the blood pressure
REFERENCES: Kozier and Erbs Fundamental of Nursing, 8th Edition, (2007), Volume 1, pp. 558-
560, 553.

Smeltzer, Suzanne C. Brunner and Suddarths Medical-Surgical Nursing 12th Edition, (2010), Volume
1, pp. 641, 707.

24. Patients suffering from COPD are taught to avoid shifts to temperature and humidity. It should be
emphasized that heat increases body temperature and thereby raising the:
A. Risk for infection C. The oxygen requirements
B. Anxiety level D. Fluid intake

ANSWER: C

RATIONALE:

Exposure to extremes of heat and cold needs to be avoided by clients who have COPD as heat
increases the body temperature, thereby raising the OXYGEN REQUIREMENTS, cold tends to
promote bronchospasms

OPTION A is INCORRECT. Bronchopulmonary infections must be controlled to diminish


inflammatory edema and to permit recovery of normal ciliary action. Minor infections that are no
consequence to people with normal lungs can be life threatening to people with COPD. Infection
compromises lung function and is common cause of respiratory failure in clients with COPD. To
decrease the risk of contracting infection, the nurse should encourage clients with COPD to be
immunized against influenza and S. pneumonia.

OPTION B is INCORRECT. Anxiety experience by COPD clients is may be due to dyspnea, change
in health status, and situational crisis. To help deal with anxiety, the nurse may provide written
instruction on what to do if flare up or worsening of condition occurs. In addition stressing the
importance of pursed-lip breathing and diaphragmatic breathing technique may help to relieve anxiety
especially during periods of panic or attack.

OPTION D is INCORRECT. High temperature level does not increase fluid intake, instead it may lead
to dehydration. In addition, clients, especially those with thick mucus accumulation should be
encouraged to increase fluid intake at least 2 to 3 L/day unless contraindicated. This intervention
helps to keep the mucus moist and easier to expectorate.

REFERENCES: Smeltzer, Suzanne C. Brunner and Suddarths Medical-Surgical Nursing 12th Edition,
(2010), Volume 1, p. 613,

Ignatavicius and Workmans Medical-Surgical Nursing: Critical Thinking for Collaborative Care 5th
Edition, (2006), Volume 1, p. 605.

25. COPD patients may be taught the following pulmonary hygiene measures to improve clearance of
airway secretion, except:
A. Effective coughing B. Measure fluid intake C. Postural drainage D.
Complete bed rest

ANSWER: D

RATIONALE:
Complete bed rest is not advisable to clients with COPD especially those with thick mucus
accumulation, as complete bed rest would further aggravates the accumulation of mucus. The nurse
should advise to client regarding alternating activates with rest periods.

OPTIONS A, B and C are INCORRECT. These options are all accurate intervention to improve
clearance of airway; however they are incorrect because the client is looking for an intervention that
will not help the problem (ineffective airway clearance).

Clients with COPD often have difficulty with removal of secretions, which result in compromised
breathing and poor oxygenation. In addition, excessive mucus increases the risk for infection to
develop. To help cope with the problem, the nurse may encourage the client to do the following
pulmonary hygiene:

Effective coughing (controlled coughing) Because client with COPD has excessive
mucus, coughing at specific times of the day is helpful. Teach the client to COUGH ON
ARISING IN THE MORING TO ELIMINATE MUCUS THAT COLLECTED DURING NIGHT.
COUGING PRIOR TO MEATIMES MAY FACILITATES A MORE PLEASANT MEAL.
COUGHING BEFORE BEDTIME MAY ENSURE CLEAR LUNGS FOR A LESS
INTERRUPTED SLEEP.
Chest physiotherapy and postural drainage Chest physiotherapy with postural drainage
helps move secretion into the central airways, re-expand the lung tissue and promote efficient
use of ventilator muscle. Chest PT combines with chest percussion with vibration to loose
secretions. Postural drainage uses specific position and gravity to help remove secretion.
Suctioning suctioning is performed ONLY when ABNORMAL BREATH SOUNDS ARE
PRESENT
Positioning Assist the client who can tolerate sitting in a chair out of bed for 1-hour period
two to three times a day. This position helps move secretions and keeps the diaphragm in a
better position for ventilation
Hydration Unless hydration needs to be avoided for other health problems, instruct the
client with COPD to drink at least 2 to 3 L/day. This intervention keeps the secretions moist,
thereby facilitating easier expectoration.
REFERENCE: Ignatavicius and Workmans Medical-Surgical Nursing: Critical Thinking for
Collaborative Care 5th Edition, (2006), Volume 1, p. 604.

SITUATION: Kelsey is a 43-year-old unrestrained driver admitted to the hospital after a motor vehicle
crash. She sustained fractured ribs in the left side, a pelvic fracture, and a left femur fracture.

26. Which assessment data would the nurse expect to see in a client with acute respiratory distress
syndrome?
A. Deficient ventilation and perfusion lung scans C. Bilateral pulmonary infiltrates on
chest x-ray
B. Increased anterior-posterior chest diameter D. Positive sputum culture

ANSWER: C

RATIONALE:
OPTION A is INCORRECT. This is for pulmonary embolism

OPTION B is INCORRECT. This manifestation is for emphysema

OPTION D is INCORRECT. ARDS is not infectious


27. Which pathologic change is responsible for the development of pulmonary edema in the patient with
acute respiratory distress syndrome?
A. Right-sided heart failure C. Extravascular fluid volume excess
B. Alveolocapillary membrane damage D. Pulmonary artery infarct

ANSWER: B

RATIONALE:

28. Kelsey, who is belligerent and confused, has the following arterial blood gas results: pH 7.55, PaO 2
68 mmHg; PCO2 48 mmHg; and HCO3- 25 mEq/L. Which statement would best explain the reason for
the clients behavior?
A. The client is frightened by his breathing difficulty.
B. Metabolic alkalosis is present and causing the changes in mental status.
C. This behavior is typical for any client experiencing an acute respiratory crisis.
D. The behavior is a result of the clients respiratory acidosis.

ANSWER: C

RATIONALE:
In any clients with acute respiratory crisis, changes in personality and consciousness is evident

OPTION B is INCORRECT. The client isnt experiencing metabolic alkalosis. In that case, this
explanation is irrelevant regarding the reason of the clients behavior

OPTION D is INCORRECT. The client is not experiencing respiratory acidosis. As you will analyzed
the ABG of the client, it does not indicate that the client is experiencing respiratory acidosis, because
the pH level is 7.55 which indicates alkalosis

29. Kelsey continues to show signs of hypoxemia. The nurse knows that the patients treatment plan is
appropriate when the doctor:
A. Prescribes mechanical ventilation C. Prescribes low FiO2
B. Prescribes chest physiotherapy D. Inserts a chest tube

ANSWER: A

RATIONALE:

30. Kelsey has been intubated and placed on a ventilator. The nurse observes increased peak pressures,
absent breath sounds in one lobe, and restlessness. The nurse should prepare for emergency testing
and treatment for which disorder?
A. Cardiac tamponade C. Heart failure
B. Pulmonary edema D. Pneumothorax

ANSWER: D

RATIONALE:

The assessment findings shown by the client is indicative of pneumothorax. Nursing intervention for
pneumothorax due to mechanical ventilator are

SITUATION: Hero, 8 years old, has two chest tubes connected to a disposable water sealed drainage
system because of chest injuries from a vehicular accident.

31. The nurse observed that the drainage from the chest tubes have not increased from the previous shift
report. Which of the following is the priority action of the nurse?
A. Change position of the patient C. Check the chest tube for kinks
B. Document observation in the chart D. Assess for breath sounds

ANSWER: C

RATIONALE:
The priority nursing action in this situation is to assess for the patency of the tube. However, make
sure to avoid milking or stripping the tube because these techniques do not improve chest tube
patency. Squeezing hand over hand along the tubing and releasing the tubing between squeezes
may help improve patency.

OPTION A is INCORRECT. Reposition promotes drainage, however before doing this intervention
the nurse should first assess for any obstruction that is present.

OPTION B is INCORRECT. This is not a normal finding specially in this kind of situation, in which
blood accumulation happens inside the pleural space because of chest injuries. The drainage should
be increase over time. However, make sure that the fluid in bottle one must NEVER come into
contact with either the draining from the client o the tube connecting this bottle to the water seal
chamber because if the fluids come in contact with the tube, drainage will stop.

OPTION D is INCORRECT. Assessment of breath sound is indicated to a client with chest tube but
not particularly I this kind of situation where in tube patency is the problem.

REFERENCES: Koziers Fundamentals of Nursing 8th Edition, (2008), Volume 2, p. 1394.

Ignatavicius and Workmans Medical-Surgical Nursing: Critical Thinking for Collaborative Care 5th
Edition, (2006), Volume 1, pp. 623 & 624.

32. Frequent assessment of the closed drainage system is important to ensure appropriate functioning.
The nurse observes that water level fluctuates with respiratory effort. The nurse considers this as a
sign of:
A. Trapped air B. An inefficient system C. Patent tubes D. Air
leaks

ANSWER: C

RATIONALE:

Fluctuation of the water level in the water seal shows effective connection between the pleural cavity
and he drainage chamber and indicates that the drainange system is PATENT.

OPTION B is INCORRECT. This observation by the nurse isnt indicative of inefficient drainage
system

OPTION D is INCORRECT. Air leaks should be suspected if the continuous bubbling is observed in
the water seal chamber.

REFERENCES: Koziers Fundamentals of Nursing 8th Edition, (2008), Volume 2, p. 1394.

Ignatavicius and Workmans Medical-Surgical Nursing: Critical Thinking for Collaborative Care 5th
Edition, (2006), Volume 1, pp. 623 & 624.

Smeltzer, Suzanne C. Brunner and Suddarths Medical-Surgical Nursing 12th Edition, (2010), Volume
1, pp. 668-670.

33. The nurse works with a nursing aide. Which of the following is a correct action of the nurse? The
nurse directed the nursing aide to:
A. Always check that clamp is available at the bedside
B. Observe regularly the amount and color of drainage from chest tubes
C. Report signs of patients discomforts at the site of the chest tubes
D. Turn the patient regularly and maintain connections of the tubes
ANSWER: A

RATIONALE:

Checking of equipment could be delegated to nursing aide.

OPTIONS B, C and D are INCORRECT. Assessment, nursing intervention and care of invasive lines
is the sole responsibility of the nurse and should not be delegated to nursing aide.

REFERENCE: Hogan, Marry Ann,. et. al. Fundamentals of Nursing 2 nd Edition, p. 461.

34. While the nurse was turning the patient during bed bath, one of the chest tubes was pulled out from
its site. Which of the following will the nurse do first?
A. Cover wound site with sterile gauze C. Reinsert the chest tube
B. Disconnect chest tube from drainage system D. Clamp the chest tube

ANSWER: A

RATIONALE:

If the tube is inadvertently pulled out, the wound should be immediately covered with a dry sterile
dressing. If you can hear air leaking out of the site, ensure that the dressing is NOT OCCLUSIVE. If
the air canot escape, this would lead to a TENSION PNEUMOTHORAX.

OPTION B is INCORRECT. Disconnecting chest tube to the drainage sysem would not help the
situation.

OPTION C is INCORRECT. Reinserting the tube would be the responsibility of the physician.

OPTION D is INCORRECT. Clamping he tube would not help the problem. Moreover, clamping
should be avoided unless specifically ordered by the clients attending physician.

REFERENCE: Koziers Fundamentals of Nursing 8th Edition, (2008), Volume 2, p. 1394.

35. To determine if chest tubes are in place and pneumothorax is corrected, which of the following will the
nurse expect physician to order?
A. Arterial blood gas analysis C. Tidal volume measurement
B. Thoracentesis D. Chest radiograph

ANSWER: D

RATIONALE:

Chest radiograph is the definitive test to determine if the chest tubes are in place or if the
pneumothorax has been corrected.

OPTION A is INCORRECT. Arterial blood gas analysis helps provide information regarding acid-base
imbalance that occurring in the body in response to a specific health problem.

OPTION B is INCORRECT. Thoracentesis is defined as aspiration of fluid in the pleural space. This
procedure is used as diagnostic and curative specially to clients with pleural effusion.

OPTION C is INCORRECT. Tidal volume measurement does not indicate if the chest tube is in place
or if the pneumothorax has been corrected. Instead this test is useful in clients with constrictive lung
disease especially those with COPD.
REFERENCE: Silvestri, Linda Anne. Saunders Comprehesive Review for the NCLEX-RN
EXAMINATION 4TH Edition, Chapter 21, p. 38

SITUATION: Degenerative neurologic disorders cause devasting physical and emotional changes in
clients. Nurses are faced with physical and psychological challenges when caring for these clients.

36. A forty year old male is brought to the hospital complaining of extreme fatigue, difficulty swallowing,
tremors and double vision. During assessment the nurse validated the presenting complaints and
notes scanning speech, muscle weakness and painful sensation. From this information, the physician
supports which of the following diagnosis?
A. Multiple Sclerosis C. Guillain Barre Syndrome
B. Parkinsons Disease D. Alzheimers Disease

ANSWER: A

RATIONALE:
The client in the situation is experiencing manifestations related to multiple sclerosis. Multiple
sclerosis (MS) is an immune-mediated, progressive demyelinating disease of the CNS. It results in
impaired transmission of nerve impulses. Clients between 20 and 40 years, females are often
affected.

OPTION B is INCORRECT. Parkinsons disease is a slowly progressing neurologic movement


disorder that eventually leads to disability. The disease affects men more often than women. . Most
cases of Parkinsons disease is unknown, research suggest several causative factors, including
genetics, athetosclerosis, excessive accumulation of oxygen free radicals, viral infections, head
trauma, and chronic use of antipsychotic medications.

OPTION C is INCORRECT. Guillain-Barre syndrome is an autoimmune attack on the peripheral


nerve myelin. The result is acute, rapid segmental demyelination of peripheral nerves and some
cranial nerves, producing ascending weakness with dyskinesia, hyporeflexia and paresthesia. An
antecedent events (most often a viral infection) precipitates clinical presentation.

OPTION C is INCORRECT. Alzeimers disease, or senile dementia of the Alzheimers disease, is a


chronic, progressive and degenerative brain disorder that is accompanied by profound effects on
memory, cognition, and ability for self care.

37. The physician notes a sign of cholinergic crisis. Which of the following clinical manifestations should
alert the nurse of this condition?
A. Restlessness, dyspnea and ineffective cough reflex
B. Cyanosis and absence of swallowing reflex
C. Abdominal cramps, diarrhea and excessive secretions
D. Hypoxia, increased in blood pressure and heart rate

ANSWER: C

RATIONALE:

Myasthenia gravis, an autoimmune disorder affecting the myoneural junction, is characterized by


varying degrees of weakness of voluntary muscles. The manifestation of the condition is being
controlled with pharmacologic administration of Pyridostigmine (Mestinon, an anticholinesterase
medication. However, over medication can lead to CHOLINERGIC CRISIS, which is manifested by
diarrhea, abdominal cramps, nausea, weakness, tachycardia, hypotension, increased bronchial
secretion, irritability and anxiety as well as head ache and sometimes insomnia.
OPTIONS A, B and D are INCORRECT. These manifestations are not related to cholinergic crisis.

REFERENCE: Smeltzer, Suzanne C. Brunner and Suddarths Medical-Surgical Nursing 12th Edition,
(2010), Volume 2, p. 1965.

38. The nurse determines that risk for injury is the priority nursing diagnosis for a client with Parkinsons
disease. Safety is the major factor. The following nursing intervention help prevent any occurrence of
falls, except:
A. Clearing floor of rugs and loose carpets C. Application of leg braces
B. Use of memory devices D. Use of canes and walkers

ANSWER: B

RATIONALE:

Use of memory device helps the cognitive ability of client with Parkinsons disease, but does not
promote safety.

OPTIONS A, C and D are INCORRECT. Clearing the floor of rugs and loose carpets, application of
leg braces and use of cane and walkers help prevents falls or injury.

39. A nurse is caring for a client with Guillain Barre Syndrome. Which of the following is most important
in the goal of care of the client during the acute phase?
A. Maintaining a balance of rest and sleep C. Preventing complications
B. Alleviating anxiety D. Dealing with depression

ANSWER: C

RATIONALE:

Guillain Barre Syndrome is an autoimmune attack on the peripheral nerve myelin. The result is
acute, rapid segmental demylination of the peripheral nerves and some cranial nerves, producing
ASCENDING WEAKNESS. Prior to the onset of manifestation (2 weeks before the occurrence of
manifestation) some client reports antecedent event most often a viral infection on the lungs and the
gastrointestinal tract. Becayuse of possibility of rapid progression and neuromuscular respiratory
failure, Guillain-Barre syndrome is a medical emergency, requiring management in an intensive care
unit. During the ACUTE PHASE of the illness, the goal of care is to minimize the effects of immobility
and occurrence of other complication like respiratory paralysis.

OPTIONS A, B and D are INCORRECT. All of these are indicated in the care of client with Guillain
Barre syndrome. However, option C is more specific goal during the acute phase of the illness.

REFERENCE: Smeltzer, Suzanne C. Brunner and Suddarths Medical-Surgical Nursing 12th Edition,
(2010), Volume 2, p. 1969.

40. An elderly female client diagnosed with Alzheimers disease is brought to the hospital by her daughter
because she could no longer recognize herself and others, is incontinent and manifests delusions
and hallucinations. Based from the clients symptoms and abilities at what stage of the disease can
the nurse classify the client?

A. Stage 2 B. Stage 1 C. Stage 3 D.


Stage 4

ANSWER: C
RATIONALE:

Alzheirmers Disease is a progressive, irreversible, degenerative neurologic disease. It usuall begins


with gradual losses of cognitive function and disturbance in behaivior and affect. Warnings signs that
may indicate the development of this condition include memory loss affecting ability to function in job,
difficulty with familiartask, problems with language and abstract thinking and changes in mood.
Alzheimers Disease can be classified according to stages. In this client, the manifestation observed
in the situation may indicate that the client is on stage 3. This stage is chracerized by complete
dependency and loss of language, loss f bowel and bladder control and progressive loss of cognitive
abilits.

OPTION A is INCORRECT. Clients on stage 2 usually manifest memory deficits, impaired language,
difficulty with motor activity, inability to carry out ADLs impaired judgement , increase agitation and
inability to write as well as asterognosis (inability to identy objects by touch)

OPTION B is INCORRECT. In stage1 client usually appears healthy and alert, cognitive deficits are
undetected with subtle personality changes. However, mild memory deficit may occur. The client may
seens to be restless and uncoordinated.

OPTION D is INCORRECT. There is no stage 4.

You might also like